You are on page 1of 92

2021

CBSE BOARD
SA MPLE PA PER

MATHEMATICS

2021
t CBSE SAMPLE PAPER AS PER THE LATEST CBSE
CBSE BOARD
SYLLABUS AND MARKING SCHEME
Sample Paper
t BASED ON NCERT COURSE STRUCTURE includes
Marking Scheme
t LAST 3 YEARS CBSE BOARD PAPERS FULLY
Issued by
SOLVED
CBSE

CLASS X
Syllabus*

COURSE STRUCTURE
MATHEMATICS (CLASS–X)

UNITS UNIT NAME MARKS


I. NUMBER SYSTEMS 06
II. ALGEBRA 20
III. COORDINATE GEOMETRY 06
IV. GEOMETRY 15
V. TRIGONOMETRY 12
VI. MENSURATION 10
VII. STATISTICS & PROBABILITY 11

TOTAL 80

UNIT I : NUMBER SYSTEMS


1. Real Numbers (8 Periods)
Fundamental Theorem of Arithmetic - statements after reviewing work done earlier and
after illustrating and motivating through examples, Proofs of irrationality of 2 , 3 , 5
Decimal representation of rational numbers in terms of terminating/non-terminating
recurring decimals.

UNIT II : ALGEBRA
1. Polynomials (4 Periods)
Zeros of a polynomial. Relationship between zeros and coefficients of quadratic polynomials.

* Issued by CBSE.

M-3

Click Here to Buy Main Book Online


M-4 MATHEMATICS—X

2. Pair of Linear Equations in Two Variables (11 Periods)


Pair of linear equations in two variables and graphical method of their solution, consistency/
inconsistency.
Algebraic conditions for number of solutions. Solution of a pair of linear equations in two
variables algebraically - by substitution, by elimination. Simple situational problems. Simple
problems on equations reducible to linear equations.
3. Quadratic Equations (10 Periods)
2
Standard form of a quadratic equation ax + bx + c = 0, (a  0). Solutions of quadratic
equations (only real roots) by factorization, and by using quadratic formula. Relationship
between discriminant and nature of roots.
4. Arithmetic Progressions (4 Periods)
Motivation for studying Arithmetic Progression Derivation of the nth term and sum of the
first n terms of A.P.

UNIT III : COORDINATE GEOMETRY


1. Lines (In two-dimensions) (10 Periods)
Review: Concepts of coordinate geometry, graphs of linear equations. Distance formula.
Section formula (internal division).

UNIT IV : GEOMETRY
1. Triangles (10 Periods)
Definitions, examples, counter examples of similar triangles.
1. (Prove) If a line is drawn parallel to one side of a triangle to intersect the other two sides
in distinct points, the other two sides are divided in the same ratio.
2. (Motivate) If a line divides two sides of a triangle in the same ratio, the line is parallel to
the third side.
3. (Motivate) If in two triangles, the corresponding angles are equal, their corresponding
sides are proportional and the triangles are similar.
4. (Motivate) If the corresponding sides of two triangles are proportional, their corresponding
angles are equal and the two triangles are similar.
5. (Motivate) If one angle of a triangle is equal to one angle of another triangle and the sides
including these angles are proportional, the two triangles are similar.
6. (Motivate) If a perpendicular is drawn from the vertex of the right angle of a right
triangle to the hypotenuse, the triangles on each side of the perpendicular are similar to
the whole triangle and to each other.
7. (Prove) In a right triangle, the square on the hypotenuse is equal to the sum of the
squares on the other two sides.
2. Circles (8 Periods)
Tangent to a circle at point of contact.
1. (Prove) The tangent at any point of a circle is perpendicular to the radius through the
point of contact.
2. (Prove) The lengths of tangents drawn from an external point to a circle are equal.

Click Here to Buy Main Book Online


SYLLABUS M-5

3. Constructions (4 Periods)
1. Division of a line segment in a given ratio (internally).
2. Tangents to a circle from a point outside it.

UNIT V : TRIGONOMETRY
1. Introduction to Trigonometry (8 Periods)
Trigonometric ratios of an acute angle of a right-angled triangle. Proof of their existence
(well defined). Values of the trigonometric ratios of 30°, 45° and 60°. Relationships between
the ratios.
2. Trigonometric Identities (10 Periods)
Proof and applications of the identity sin2A + cos2A = 1. Only simple identities to be given.
3. Heights and Distances: Angle of elevation, Angle of Depression. (8 Periods)
Simple problems on heights and distances. Problems should not involve more than two
right triangles. Angles of elevation / depression should be only 30°, 45°, 60°.

UNIT VI : MENSURATION
1. Areas Related to Circles (10 Periods)
Motivate the area of a circle; area of sectors and segments of a circle. Problems based on
areas and perimeter / circumference of the above said plane figures. (In calculating area of
segment of a circle, problems should be restricted to central angle of 60°and 90° only. Plane
figures involving triangles, simple quadrilaterals and circle should be taken.)
2. Surface Areas and Volumes (8 Periods)
(i) Surface areas and volumes of combinations of any two of the following: cubes, cuboids,
spheres, hemispheres and right circular cylinders/cones.
(ii) Problems involving converting one type of metallic solid into another and other mixed
problems. (Problems with combination of not more than two different solids be taken).

UNIT VII : STATISTICS AND PROBABILITY


1. Statistics (10 Periods)
Mean, median and mode of grouped data (bimodal situation and step deviation method for
finding the mean to be avoided).
2. Probability (10 Periods)
Classical definition of probability. Simple problems on finding the probability of an event.

Click Here to Buy Main Book Online


M-6 MATHEMATICS—X

MATHEMATICS-STANDARD
QUESTION PAPER DESIGN
CLASS – X (2020-21)
Time : 3 Hours Max. Marks: 80

S.No. Typology of Questions Total Marks % Weightage (approx.)

1. Remembering: Exhibit memory of previously learned 43 54


material by recalling facts, terms, basic concepts,
and answers.
Understanding: Demonstrate understanding of facts
and ideas by organizing, comparing, translating,
interpreting, giving descriptions, and stating main ideas

2. Applying: Solve problems to new situations by applying 19 24


acquired knowledge, facts, techniques and rules in a
different way.

3. Analysing : Examine and break information into parts


by identifying motives or causes. Make inferences and
find evidence to support generalizations
Evaluating: Present and defend opinions by making 18 22
judgments about information, validity of ideas, or
quality of work based on a set of criteria.
Creating: Compile information together in a different
way by combining elements in a new pattern or
proposing alternative solutions

Total 80 100

INTERNAL ASSESSMENT 20 Marks


 Pen Paper Test and Multiple Assessment (5+5) 10 Marks
 Portfolio 05 Marks
 Lab Practical (Lab activities to be done from the prescribed books) 05 Marks

Click Here to Buy Main Book Online


MRP: 575.00

Click Here to Buy the Book Online

Committed to Educate the Nation NEW AGE INTERNATIONAL (P) LIMITED, PUBLISHERS
LONDON • NEW DELHI • NAIROBI

IN INDIA Visit us at www.newagegolden.com


Contents

1. Syllabus (Code No. 041) M-3


1. CBSE Sample Question Paper – 2020-21 with Marking Scheme M-7
2. CBSE Board Examination Paper – 2020 (Solved) M-22
3. CBSE Board Examination Paper – 2019 (Solved) M-40
4. CBSE Board Examination Paper – 2018 (Solved) M-62

Click Here to Buy Main Book Online


MRP: 575.00

Click Here to Buy the Book Online

Committed to Educate the Nation NEW AGE INTERNATIONAL (P) LIMITED, PUBLISHERS
LONDON • NEW DELHI • NAIROBI

IN INDIA Visit us at www.newagegolden.com


CBSE SAMPLE QUESTION PAPER 2020–21
Mathematics–Standard (041)
(With Marking Scheme)
Class–X

Time Allowed: 3 hours Maximum Marks: 80

General Instructions:
1. This question paper contains two parts A and B.
2. Both Part A and Part B have internal choices.
Part-A
1. It consists of two Sections I and II.
2. Section I has 16 questions of 1 mark each. Internal choice is provided in 5 questions.
3. Section II has 4 questions on case study. Each case study has 5 case-based sub-parts. An examinee is to
attempt any 4 out of 5 sub-parts.
Part-B
1. Question No. 21 to 26 are Very Short Answer Type Questions of 2 marks each.
2. Question No. 27 to 33 are Short Answer Type Questions of 3 marks each.
3. Question No. 34 to 36 are Long Answer Type Questions of 5 marks each.
4. Internal choice is provided in 2 questions of 2 marks, 2 questions of 3 marks and 1 question of 5 marks.

PART–A
Section I has 16 questions of 1 mark each. Internal choice is provided in 5 questions.
1. If xy = 180 and HCF (x, y) = 3, then find the LCM(x, y). (1)

OR


The decimal representation of will terminate after how many decimal places?
 ª  
Sol. Product of numbers = LCM(x, y) × HCF(x, y)
⇒ LCM (x, y) × (3) = 180
⇒ LCM (x, y) = 60

OR

  


Here = = = 11.6696
 ×   ×  
∴ Four decimal places

M-7

Click Here to Buy Main Book Online


M-8 CBSE SAMPLE QUESTION PAPER–2020–21
2. If the sum of the zeroes of the quadratic polynomial 3x2 – kx + 6 is 3, then find the value of k.
Sol. If  and  are sum of the zeroes of 3x2 – kx + 6 then (1)

 ( k) k
 = 
3 3

k
 3=
3
 k=9
3. For what value of k, the pair of linear equations 3x + y = 3 and 6x + ky = 8 does not have a solution.
Sol. The given pair of linear equations does not have a solution if (1)
a1 b c
= 1  1
a2 b2 c2
3 1 3 3 1
 =   =  k=2
6 k 8 6 k
4. If 3 chairs and 1 table costs ` 1500 and 6 chairs and 1 table costs ` 2400. Form linear equations to
represent this situation. (1)
Sol. Let the cost of 1 chair = ` x
And the cost of 1 table = ` y
Now according to the question,
3x + y = 1500 and 6x + y = 2400
5. Which term of the AP 27, 24, 21, .... is zero? (1)

OR
In an Arithmetic Progression, if d = – 4, n = 7, an = 4, then find a.
Sol. Let nth term of the given A.P. is zero then
an = a + (n – 1)d
 0 = 27 + (n – 1) (– 3) [ a = 27, d = 24 – 27 = – 3]
 0 = 27 – 3n + 3
 30 = 3n  n = 10

OR
nth term of an AP is given by
an = a + (n – 1)d
 4 = a + (7 – 1) × (– 4)
 4 = a + 6 × (– 4)  a = – 28
6. For what values of k, the equation 9x2
+ 6kx + 4 = 0 has equal roots? (1)
Sol. The given quadratic equation 9x + 6kx + 4 = 0 has equal roots then b2 – 4ac = 0
2

(6k)2 – 4 × 9 × 4 = 0
 36k2 = 144  k2 = 4  k=±2
7. Find the roots of the equation x2 + 7x + 10 = 0 (1)

OR
For what value(s) of ‘a’ quadratic equation 3ax2 – 6x + 1 = 0 has no real roots?

Click Here to Buy Main Book Online


MATHEMATICS–X M-9
Sol. The given equation is
x2 + 7x + 10 = 0 ⇒ x2 + 5x + 2x + 10 = 0
⇒ x(x + 5) + 2(x + 5) = 0 ⇒ (x + 5) (x + 2) = 0
⇒ x = – 5, x = – 2

OR
The given equation 3ax2 – 6x + 1 = 0 has no real roots i.e. b2 – 4ac < 0
⇒ (– 6)2 – 4(3a) (1) < 0 ⇒ 36 – 12a < 0
⇒ 12a > 36 ⇒ a>3
8. If PQ = 28 cm, then find the perimeter of DPLM. (1) P

Sol. Here,
PQ = PT L N M
⇒ PL + LQ = PM + MT ⇒ PL + LN = PM + MN
O T
Perimeter (ΔPLM) = PL + LM + PM
= PL + LN + MN + PM
= 2(PL + LN) = 2(PL + LQ)
= 2 × 28 = 56 cm
9. If two tangents inclined at 60° are drawn to a circle of radius 3 cm then find the length of each
tangent. (1)

OR
PQ is a tangent to a circle with centre O at point P. If DOPQ is an isosceles triangle, then find
РOQP.
Sol. According to given condition in question A
PA and PB are two tangents drawn to a circle then

3
cm
In Δ PAO T
O
tan 30° = AO/PA 30°
P 30°
⇒    = 3/PA

⇒ PA =   EO = PB B

OR P
In Δ OPQ
Q
∠ P + ∠ Q + ∠ O = 180°
⇒ 2 ∠ Q + ∠ P = 180° (Q Δ OPQ is an isosceles triangle) O

⇒ 2 ∠ Q + 90° = 180°


⇒ 2 ∠ Q = 90° ⇒ ∠ Q = 45°
10. In the DABC, D and E are points on side AB and AC respectively such that DE || BC. If AE = 2 cm,
AD = 3 cm and BD = 4.5 cm then find CE. (1)
Sol. According to mid-point theorem A

#& #'
=
$& %'
  D E
∴ =
 %'
 × 
⇒ CE = = 1.5 × 2 ⇒ CE = 3 cm
 B C

Click Here to Buy Main Book Online


M-10 CBSE SAMPLE QUESTION PAPER–2020–21
11. In the figure, if B1, B2, B3 ..... and A1, A2, A3, ..... have been marked at equal distances. In what
ratio C divides AB? (1)
Y B5 B4 B3 B2 B1
B

A
A1 A2 A3 A4 A5 A6 A7 A8 X

Sol. 8.5
12. sin A + cos B = 1, A = 30° and B is an acute angle, then find the value of B. (1)
Sol. Given sin 30° + cos B = 1

  
⇒ + cos B = 1 ⇒ cos B =  − =
  
⇒ cos B = cos 60° ⇒ B = 60°
13. If x = 2 sin2 q and y = 2 cos2 q + 1, then find x + y. (1)
Sol. x + y = 2 sin2 θ + 2 cos2 θ + 1
= 2 (sin2 θ + cos2 θ) + 1 [Q (sin2 θ + cos2 θ = 1)]
=3
14. In a circle of diameter 42 cm, if an arc subtends an angle at 60° at the centre where p = 22/7, then
what will be the length of arc. (1)
Sol. Length of arc = θ/360° (2πr) = 60/360 (2 × 22/7 × 21) = 22 cm
15. 12 solid spheres of the same radii are made by melting a solid metallic cylinder of base diameter
2 cm and height 16 cm. Find the diameter of the each sphere. (1)
Sol. According to question, πR2H = 12 × 4/3πr3 where r is radius of sphere
1 × 1 × 16 = 4/3 × r3 × 12
⇒ r3 = 1 ⇒ r=1
∴ d = 2 × 1 = 2 cm
16. Find the probability of getting a doublet in a throw of a pair of dice. (1)

OR
Find the probability of getting a black queen when a card is drawn at random from a well-
shuffled pack of 52 cards.

Sol. Probability of getting a doublet =

OR
 
Probability of getting a black queen = =
 

Click Here to Buy Main Book Online


MATHEMATICS–X M-11
Section-II: Case study based questions are compulsory. Attempt any four subparts of each question. Each subpart
carries 1 mark.
17. Case Study based-1
SUN ROOM
The diagrams show the plans for a sun room. It will be built onto the wall of a house. The four
walls of the sunroom are square clear glass panels. The roof is made using
• Four clear glass panels, trapezium in shape, all the same size
• One tinted glass panel, half a regular octagon in shape

Y
A B C D

J H
I

G E
F
B
Top view
B

1 cm
A
A
P Q S

Front view
Not to scale O Scale 1 cm = 1 m X

(a) Refer to Top View


Find the mid-point of the segment joining the points J(6, 17) and I (9, 16). (1)
(i) (33/2, 15/2) (ii) (3/2, 1/2) (iii) (15/2, 33/2) (iv) (1/2, 3/2)
(b) Refer to Top View
The distance of the point P from the y-axis is (1)
(i) 4 (ii) 15 (iii) 19 (iv) 25
(c) Refer to Front View
The distance between the points A and S is (1)
(i) 4 (ii) 8 (iii) 16 (iv) 20
(d) Refer to Front View
Find the co-ordinates of the point which divides the line segment joining the points A and B in
the ratio 1 : 3 internally. (1)
(i) (8.5, 2.0) (ii) (2.0, 9.5) (iii) (3.0, 7.5) (iv) (2.0, 8.5)
(e) Refer to Front View
If a point (x, y) is equidistant from the Q(9, 8) and S(17, 98) then (1)
(i) x + y = 13 (ii) x – y = 0 (iii) y – 13 = 0 (iv) x – y = 13

⎛   ⎞
Sol. (a) (iii) ⎜  ⎟ (b) (i) 4 (c) (iii) 16 (d) (iv) (2.0, 8.5) (e) (ii) x – 13 = 0
⎝   ⎠

Click Here to Buy Main Book Online


M-12 CBSE SAMPLE QUESTION PAPER–2020–21
18. Case Study Based-2
SCALE FACTOR AND SIMILARITY
SCALE FACTOR
A scale drawing of an object is the same shape as the object but a
different size.
The scale of a drawing is a comparison of the length used on a drawing
to the length it represents. The scale is written as a ratio.
SIMILAR FIGURES
The ratio of two corresponding sides in similar figures is called the
scale factor.
NGPIVJ KP KOCIG
Scale factor =
EQTTGURQPFKPI NGPIVJ KP QDLGEV

If one shape can become another using Resizing then the shapes are similar.

Rotation or Turn Reflection or Flip Translation or Slide

Hence the two shapes are similar when one can become the other after a resize, flip, slide or turn.
(a) A model of a boat is made on the scale of 1 : 4. The model is 120 cm long. The full size of the
boat has a width of 60 cm. What is the width of the scale model? (1)

(i) 20 cm (ii) 25 cm (iii) 15 cm (iv) 240 cm


(b) What will effect the similarity of any two polygons? (1)
(i) They are flipped horizontally (ii) They are dilated by a scale factor
(iii) They are translated down (iv) They are not the mirror image of one another.
(c) If two similar triangles have a scale factor of a : b. Which statement regarding the two triangles
is true? (1)
(i) The ratio of their perimeters is 3a : b
(ii) Their altitudes have a ratio a : b

C
(iii) Their medians have a ratio D

(iv) Their angle bisectors have a ratio a2 : b2

Click Here to Buy Main Book Online


MATHEMATICS–X M-13
(d) The shadow of a stick 5 m long is 2 m. At the same time the shadow of a tree 12.5 m high is
(1)

Tree

Stick

Shadow
Shadow

(i) 3 m (ii) 3.5 m (iii) 4.5 m (iv) 5 m


(e) Below you see a student’s mathematical model of a farmhouse roof with measurements. The
attic floor, ABCD in the model, is a square. The beams that support the roof are the edges of a
rectangular prism, EFGHKLMN. E is the middle of AT, F is the middle of BT, G is the middle
of CT, and H is the middle of DT. All the edges of the pyramid in the model have length of
12 m. What is the length of EF, where EF is one of the horizontal edges of the block? (1)

12 m
H G

E F
D C
N M
K L 12 m
A 12 m B

(i) 24 m (ii) 3 m (iii) 6 m (iv) 10 m


Sol. (a) (iii) 15 cm
(b) (iv) They are not the mirror image of one another
(c) (ii) Their altitudes have a ratio a : b
(d) (iv) 5 m
(e) (iii) 6 m
19. Case Study Based-3
Applications of Parabolas-Highway Overpasses/Underpasses
A highway underpass is parabolic in shape.

Shape of cross slope


x

y
n
1 in

B1
2
(a) Parabolic Camber y = 2x /nw

Click Here to Buy Main Book Online


M-14 CBSE SAMPLE QUESTION PAPER–2020–21
Parabola: A parabola is the graph that results from p(x) = ax2 + bx + c. Parabolas are symmetric
about a vertical line known as the Axis of Symmetry. The Axis of Symmetry runs through the
maximum or minimum point of the parabola which is called the Vertex.

Axis

Axis
Vertex

Vertex

(a) If the highway overpass is represented by x2 – 2x – 8. Then its zeroes are (1)
(i) (2, – 4) (ii) (4, – 2) (iii) (– 2, – 2) (iv) (– 4, – 4)
(b) The highway overpass is represented graphically.
Zeroes of a polynomial can be expressed graphically. Number of zeroes of polynomial is equal
to number of points where the graph of polynomial (1)
(i) Intersects x-axis (ii) Intersects y-axis
(iii) Intersects y-axis or x-axis (iv) None of the above
(c) Graph of a quadratic polynomial is a (1)
(i) straight line (ii) circle (iii) parabola (iv) ellipse
(d) The representation of Highway Underpass whose one zero is 6 and sum of the zeroes is 0, is
(i) x2 – 6x + 2 (ii) x2 – 36 (iii) x2 – 6 (iv) x2 – 3 (1)
2
(e) The number of zeroes that polynomial f(x) = (x – 2) + 4 can have is (1)
(i) 1 (ii) 2 (iii) 0 (iv) 3
Sol. (a) (ii) (4, – 2) (b) (i) intersects x-axis (c) (iii) parabola
2
(d) (ii) x – 36 (e) (iii) 0
20. Case Study Based-4
100 m Race
A stopwatch was used to find the time that it took a group of
students to run 100 m.

Time (in sec.) 0–20 20–40 40–60 60–80 80–100


No. of students 8 10 13 6 3

(a) Estimate the mean time taken by a student to finish the race. (1)
(i) 54 (ii) 63 (iii) 43 (iv) 50
(b) What will be the upper limit of the modal class? (1)
(i) 20 (ii) 40 (iii) 60 (iv) 80
(c) The construction of cummulative frequency table is useful in determining the (1)
(i) Mean (ii) Median (iii) Mode (iv) All of the above
(d) The sum of lower limits of median class and modal class is (1)
(i) 60 (ii) 100 (iii) 80 (iv) 140

Click Here to Buy Main Book Online


MATHEMATICS–X M-15
(e) How many students finished the race within 1 minute? (1)
(i) 18 (ii) 37 (iii) 31 (iv) 8
Sol. (a) (iii) 43 (b) (iii) 60 (c) (ii) Median (d) (iii) 80 (e) (iii) 31

PART–B
All questions are compulsory. In case of internal choices, attempt any one.
21. 3 bells ring at an interval of 4, 7 and 14 minutes. All three bells rang at 6 am, when the three bells
will ring together next? (2)
Sol. Here 4 = 2 × 2
7=7×1
14 = 2 × 7
LCM of 4, 7, 14 = 2 × 2 × 7 = 28
∴ The three bells will ring together again at 6 : 28 am.
22. Find the point on x-axis which is equidistant from the points (2, – 2) and (– 4, 2). (2)
OR
P(– 2, 5) and Q(3, 2) are two points. Find the co-ordinates of the point R on PQ such that
PR = 2QR.
Sol. Let P(x, 0) be a point on x-axis then
PA = PB ⇒ PA2 = PB2
⇒ (x – 2)2 + (0 + 2)2 = (x + 4)2 + (0 – 2)2
⇒ x2 + 4 – 4x + 4 = x2 + 16 + 8x + 4
⇒ – 4x + 4 = 8x + 16 ⇒ x=–1
⇒ P (– 1, 0)
OR
Given PR = 2QR
PR : QR = 2 : 1
⎛ 
−  + 
 
 + 
 ⎞
4⎜ 
⎝ +  +  ⎟⎠

⎛ ⎞
4 ⎜  ⎟
⎝ ⎠

23. Find a quadratic polynomial whose zeroes are  −   and  +   . (2)


Sol. Here

Sum of zeroes =  −   +  +   = 

and Product of zeroes =


 −  
 +   = 
Required polynomial = x2 – (Sum of zeroes)x + product of zeroes
⇒ P(x) = x2 – 10x + 7

Click Here to Buy Main Book Online


M-16 CBSE SAMPLE QUESTION PAPER–2020–21
24. Draw a line segment AB of length 9 cm. With A and B as centres, draw circles of radius 5 cm and
3 cm respectively. Construct tangents to each circle from the centre of the other circle. (2)
Sol.

C A

Q P
S O R

D B

25. If tan A = 3/4, find the value of 1/sin A + 1/cos A. (2)

OR

If  sin q – cos q = 0 and 0° < q < 90°, find the value of q.


 M
Sol. tan A = =
 M
M  M 
sin A = = , cos A = =
M  M 
   
 +  
+ = + = =
UKP # EQU #    
OR
UKP θ 
 UKP θ = cos θ ⇒ =
EQU θ 

⇒ tan θ = ⇒ θ = 30°

C
26. In the figure, quadrilateral ABCD is circumscribing a circle with centre O
and AD ^ AB. If radius of incircle is 10 cm, then the value of x is (2)
Sol. ∠ A = ∠ OPA = ∠ OSA = 90°
cm
27

Hence, ∠ SOP = 90°


Also, AP = AS
38 cm
Hence, OSAP is a square R

AP = AS = 10 cm D
CR = CQ = 27 cm S O Q
10 cm
BQ = BC – CQ = 38 – 27 = 11 cm
BP = BQ = 11 cm
x = AB = AP + BP = 10 + 11 = 21 cm A P B
x cm
27. Prove that  −  is irrational, given that  is irrational. (3)
Sol. Let  −  be a rational number
We can find co-prme a and b(b ≠ 0) such that
C C
−  = ⇒ − = 
D D

Click Here to Buy Main Book Online


MATHEMATICS–X M-17

C − D
So we get, = 
D

C − D
Since a and b are integers, we get is irrational and so  is rational. But  is an irrational
D
number which contradicts our statement.

Therefore  −  is irrational.

28. If one root of the quadratic equation 3x2 + px + 4 = 0 is 2/3, then find the value of p and the other
root of the equation. (3)
OR
The roots α and β of the quadratic equation x2 – 5x + 3(k – 1) = 0 are such that a – b = 1. Find the
value of k.

Sol. Given one root of equation is then

3x2 + px + 4 = 0

⎛⎞ ⎛⎞
⎜ ⎟ + R⎜ ⎟ +  = 0
⎝⎠ ⎝⎠
 R
+ + =0 ⇒ p=–8
 
Now 3x2 – 8x + 4 = 0 ⇒ 3x2 – 6x – 2x + 4 = 0

⇒ x= or x=2

Hence x = 2.
OR

− 
Here sum of roots α + β = =5 …(1) and α–β=1 (given) …(2)

Solving (1) and (2), we get α = 3 and β=2
Now product of roots αβ = 6 or 3(k – 1) = 6
⇒ k–1=2 ⇒ k=3
29. In the figure, ABCD is a square of side 14 cm. Semi-circles are drawn with each side of square as
diameter. Find the area of the shaded region. (3)

Sol. Area of 1 segment = area of sector – area of triangle A B

= ⎛⎜ ° ⎞⎟ πT  −  ×  × 
⎝ ° ⎠ 
  
×= ×  − ×  ×  = 14 cm2
  
Area of 8 segments = 8 × 14 = 112 cm2
D C
Area of the shaded region = 14 × 14 – 112 = 196 – 112 = 84 cm2
(each petal is divided into 2 segments)
30. The perimeters of two similar triangles are 25 cm and 15 cm respectively. If one side of the first
triangle is 9 cm, find the length of the corresponding side of the second triangle. (3)
OR
In an equilateral triangle ABC, D is a point on side BC such that BD = 1/3 BC. Prove that
9AD2 = 7AB2.

Click Here to Buy Main Book Online


M-18 CBSE SAMPLE QUESTION PAPER–2020–21
Sol. If ABC and DEF are two similar triangles then
2GTKOGVGT
Δ #$% #$ $% %# #$
= =
2GTKOGVGT
Δ &'( &' '( (& &'
 
⇒ = ⇒ x = 5.4 cm
 Z
⇒ DE = 5.4 cm
OR
Construction: Draw AM ⊥ BC
  A
BD ⊥ BC, BM = BC
 
In Δ ABM AB2 = AM2 + BM2 = AM2 + (BD + BM)2
= AM2 + DM2 + BD2 + 2BD · DM
= AD2 + BD2 + 2BD(BM – BD)

= AD2 + ⎛⎜ $% ⎞⎟ +  | $% | ⎛⎜ $% − $% ⎞⎟ B
D M
C
⎝  ⎠  ⎝   ⎠
$% #$
= #& = #&
 
Hence, 7AB2 = 9AD2
31. The median of the following data is 16. Find the missing frequencies a and b if the total of the
frequencies is 70. (3)

Class 0–5 5–10 10–15 15–20 20–25 25–30 30-35 35–40


Frequency 12 a 12 15 b 6 6 4

Sol. Class Frequency Cumulative frequency

0–5 12 12
5–10 a 12 + a
10–15 12 24 + a
15–20 15 39 + a
20–25 b 39 + a + b
25–30 6 45 + a + b
30–35 6 51 + a + b
35–40 4 55 + a + b

Total 70

⇒ 55 + a + b = 70 ⇒ a + b = 15
0
− EH
⇒ Median = +  ×J
H
 −  − C
 − C
⇒ 16 =  × ⇒ 1= ⇒ a=8
 
and 55 + a + b = 70 ⇒ 55 + 8 + b = 70 ⇒ b=7

Click Here to Buy Main Book Online


MATHEMATICS–X M-19
32. If the angles of elevation of the top of the candle from A
two coins distant ‘a’ cm and ‘b’ cm (a > b) from its base
and in the same straight line from it are 30° and 60°,
q
then find the height of the candle. (3) C B
Sol. Let AB = candle
C and D are coins
Now in Δ ABC,
#$ J
tan 60° = =
$% D
A
J
⇒  = ⇒ h= D  …(1)
D
#$ J
tan 30° = = hm
$& C
 J C 60°
⇒ = ⇒ H= …(2) 30°
 C  D
C
B
b
Multiplying (1) and (2), we get
C
H2 = D  ×

H2 = ba
H2 = CD O
33. The mode of the following data is 67. Find the missing frequency x. (3)

Class 40–50 50–60 60–70 70–80 80–90


Frequency 5 x 15 12 7

H − H
Sol. Mode = + + ×J
 H − H − H
 − Z
⇒ 67 =  + × 
 −  − Z
 − Z
⇒ 7= × 
 − Z
⇒ 7 × (18 – x) = 10(15 – x) ⇒ 126 – 7x = 150 – 10x
⇒ 3x = 150 – 126 ⇒ 3x = 24
⇒ x=8
34. The two palm trees are of equal heights and are standing opposite each other on either side of the
river, which is 80 m wide. From a point O between them on the river the angles of elevation of the
top of the trees are 60° and 30°, respectively. Find the height of the trees and the distances of the
point O from the trees. (5)
OR
The angles of depression of the top and bottom of a building 50 meters high as observed from the
top of a tower are 30° and 60° respectively. Find the height of the tower, and also the horizontal
distance between the building and the river.
Sol. Let BD = river
AB = CD = palm trees = h
BO = x
OD = 80 – x

Click Here to Buy Main Book Online


M-20 CBSE SAMPLE QUESTION PAPER–2020–21
In Δ ABO
J
tan 60° =
Z
J
 =
Z
A
h= Z …(1)
In ΔCDO C

J
tan 30° =

 − Z
 J
⇒ = …(2)

 − Z B
60° 30°
D
O
Solving (1) and (2), we get 80 m
x = 20
h= Z = 34.6
The height of the trees = h = 34.6 m
BO = x = 20 m
DO = 80 – x = 80 – 20 = 60 m
OR
Let AB = Building of height 50 m X R
RT = tower of height h m 60°
30°

BT = AS = x m (h – 50) m
AB = ST = 50 m
A 30°
RS = TR – TS = (h – 50)m xm
S

45
In ΔARS, tan 30° = hm
#5

J −  50 m 50 m
= ...(1)
 Z
46
In ΔRBT, tan 60° = 60°
$6 B T
J xm
 = ...(2)
Z
Solving (1) and (2), we get
h = 75
From (2),
J 
x= = =  
 
Hence, height of the tower = h = 75 m
Distance between the building and the tower =   = 43.25 m
35. Water is flowing through a cylindrical pipe of internal diamter 2 cm, into a cylindrical tank of
base radius 40 cm at the rate of 0.7 m/sec. By how much will the water rise in the tank in half an
hour? (5)

Click Here to Buy Main Book Online


MATHEMATICS–X M-21
Sol. For pipe, r = 1 cm
Length of water flowing in 1 sec, h = 0.7 m = 7 cm
Cylindrical tank, R = 40 cm, rise in water level = H
Volume of water flowing in 1 sec = πr2h = π × 1 × 1 × 70 = 70π
Volume of water flowing in 60 sec = 70π × 60
Volume of water flowing in 30 minutes = 70π × 60 × 30
Volume of water in tank = πr2h = π × 40 × 40 × H
Volume of water in tank = Volume of water flowing in 30 minutes
π × 40 × 40 × H = 70π × 60 × 30
H = 78.75 m
36. A motorboar covers a distance of 16 km upstream and 24 km downstream in 6 hours. In the same
time it covers a distance of 12 km upstream and 36 km downstream. Find the speed of the boat in
still water and that of the stream. (5)
Sol. Let speed of the boat in still water = x km/hr and
Speed of the current = y km/hr
Downstream speed = (x + y) km/hr
Upstream speed = (x – y) km/hr
According to question
   
+ =6 …(1) and + =6 …(2)
Z+ [ Z− [ Z+ [ Z− [

 
Let = u and =v
Z+ [ Z− [
Then 24u + 16v = 6
⇒ 12u + 8v = 3 ...(3)
and 36u + 12v = 6
⇒ 6u + 2v = 1 ...(4)
Multiplying (4) by 4, we get
24u + 8v = 4v ...(5)
Subtracting (3) by (5), we get


12u = 1 ⇒ u=



Putting the value of u in (4), we get v=


   
Now = and =
Z+ [  Z− [ 
⇒ x + y = 12 and x–y=4
Thus, speed of the boat in still water = 8 km/hr,
and Speed of the current = 4 km/hr.

Click Here to Buy Main Book Online


MRP: 575.00

Click Here to Buy the Book Online

Committed to Educate the Nation NEW AGE INTERNATIONAL (P) LIMITED, PUBLISHERS
LONDON • NEW DELHI • NAIROBI

IN INDIA Visit us at www.newagegolden.com


Series : JBB/2 SET-2
Code No. 30/2/2

CBSE BOARD EXAMINATION PAPER–2020


Mathematics–X (Solved)

Time Allowed: 3 hours Maximum Marks: 80


General Instructions:
Read the following instructions very carefully and strictly follow them:
(i) This question paper comprises four sections – A, B, C and D. The question paper carries 40 questions. All
questions are compulsory.
(ii) Section A – Question no. 1 to 20 comprises of 20 questions of one mark each.
(iii) Section B – Question no. 21 to 26 comprises of 6 questions of two marks each.
(iv) Section C – Question no. 27 to 34 comprises of 8 questions of three marks each.
(v) Section D – Question no. 35 to 40 comprises of 6 questions of four marks each.
(vi) There is no overall choice in the question paper. However, an internal choice has been provided in 2 questions
of one mark, 2 questions of two marks, 3 questions of three marks and 3 questions of four marks. You have to
attempt only one of the choices in such questions.
(vii) In addition to this, separate instructions are given with each section and question, wherever necessary.
(viii) Use of calculators is not permitted.

SECTION–A
Question numbers 1 to 10 are multiple choice questions of 1 mark each. Select the correct option.
1. The value of k for which the system of linear equations x + 2y = 3, 5x + ky + 7 = 0 is inconsistent is
(1)

 
(a) − (b) (c) 5 (d) 10
 
Sol. (d) 10
2. The zeroes of the polynomial x2 – 3x – m(m + 3) are (1)
(a) m, m + 3 (b) – m, m + 3 (c) m, – (m + 3) (d) – m, – (m + 3)
Sol. (b) – m, m + 3
3. Euclid’s division Lemma states that for two positive integers a and b, there exist unique integers
q and r satisfying a = bq + r, and (1)
(a) 0 < r < b (b) 0 < r ≤ b (c) 0 ≤ r < b (d) 0 ≤ r ≤ b
Sol. (c) 0 ≤ r < b
4. The sum of exponents of prime factors in the prime-factorisation of 196 is (1)
(a) 3 (b) 4 (c) 5 (d) 2
Sol. (b) 4
5. If the point P(6, 2) divides the line segment joining A(6, 5) and B(6, y) in the ratio 3 : 1, then the
value of y is (1)
(a) 4 (b) 3 (c) 2 (d) 1
Sol. (d) 1
M-22

Click Here to Buy Main Book Online


MATHEMATICS–X M-23
6. The co-ordinates of the point which is reflection of point (– 3, 5) in x axis are (1)
(a) (3, 5) (b) (3, – 5) (c) (– 3, – 5) (d) (– 3, 5)
Sol. (c) (– 3, – 5)
7. The point P on x-axis equidistant from the points A(– 1, 0) and B(5, 0) is (1)
(a) (2, 0) (b) (0, 2) (c) (3, 0) (d) (2, 2)
Sol. (a) (2, 0)
8. The nth term of the AP a, 3a, 5a … is (1)
(a) na (b) (2n – 1)a (c) (2n + 1)a (d) 2na
Sol. (b) (2n – 1)a

  − R  − R
9. The common difference of the AP   , … is (1)
R R R
 
(a) 1 (b) (c) – 1 (d) −
R R
Sol. (c) – 1
10. The roots of the quadratic equation x2 – 0.04 = 0 are (1)
(a) ± 0.2 (b) ± 0.02 (c) 0.4 (d) 2
Sol. (a) ± 0.2
In Q.Nos. 11 to 15, fill in the blanks. Each question is of 1 mark:
11. In Fig., the angles of depressions from the observing positions O1 and O2 respectively of the object
A are _________ (1)
O1
O2
60°

45°
A
B C

Sol. 30°, 45° A

CT
' #/0
12. In Fig., MN || BC and AM : MB = 1 : 2, then _________ (1)
CT
' #$%
M N

Sol.


B C

13. In the given Fig., the length PB = _________ cm (1)


Sol. 4 cm
A 5 cm
14. In D ABC, AB =   EO , AC = 12 cm and BC = 6 cm, then РB = _________ O
(1) cm
3
P
Sol. 90°
OR
Two triangles are similar if their corresponding sides are _________ B
Proportional

Click Here to Buy Main Book Online


M-24 CBSE BOARD EXAMINATION PAPER–2020 (SOLVED)
15. The value of sin 23° cos 67° + cos 23° sin 67° is _________ (1)
Sol. 1

Q. Nos. 16 to 20 are short answer type questions of 1 mark each.


16. In Fig. is a sector of circle of radius 10.5 cm. Find the perimeter of the sector. A B

⎛  ⎞
⎜ 6CMG S  ⎟ (1)
⎝  ⎠
60°
Sol. For sector OAB
Radius (r) = 10.5 cm O
Angle at the centre O(θ) = 60°
∴ Length of the arc AB

θ
= πT
°

 °
= × ×  × = 11 cm
 °
∴ Perimeter of the sector OAB
= OA + OB + Length of Arc AB
= 10.5 + 10.5 + 11 = 32 cm
17. If a number x is chosen at random from the numbers – 3, – 2, – 1, 0, 1, 2, 3, then find the probability
of x2 < 4. (1)
OR
What is the probability that a randomly taken leap year has 52 Sundays?
Sol. Let E be the event of choosing a number x such that x2 < 4
Total number of numbers = 7
∴ Number of all possible outcomes = 7
Number of outcomes favourable to E (– 1, 0, 1) = 3

0WODGT QH QWVEQOGU HCXQWTCDNG VQ ' 


∴ P(E) = =
0WODGT QH CNN RQUUKDNG QWVEQOGU 
OR
Total number of days in a leap year = 366 = 364 + 2 = 52 weeks + 2 days
52 weeks will essentially have 52 sundays.
The remaining 2 days may be any one of the following:
(i) Sunday and Monday (ii) Monday and Tuesday
(iii) Tuesday and Wednesday (iv) Wednesday and Thursday
(v) Thursday and Friday (vi) Friday and Saturday
(vii) Saturday and Sunday
∴ Number of all possible outcomes = 7
For a randomly taken leap year to have 52 sundays, the outcomes (ii), (iii), (iv), (v) and (vi) are favourable.
∴ Number of favourable outcomes = 5
Let E be the event that a randomly taken leap year has 52 Sundays. Then,

0WODGT QH QWVEQOGU HCXQWTCDNG VQ ' 


P(E) = =
0WODGT QH CNN RQUUKDNG QWVEQOGU 

Click Here to Buy Main Book Online


MATHEMATICS–X M-25
18. A die is thrown once. What is the probability of getting a prime number? (1)
Sol. Number of all possible outcomes (1, 2, 3, 4, 5, 6) when a die is thrown once = 6
Let E be the event of getting a prime number. Then, number of outcomes (2, 3, 5) of gettng a prime number
=3

0WODGT QH QWVEQOGU HCXQWTCDNG VQ '  


∴ P(E) = = =
0WODGT QH CNN RQUUKDNG QWVEQOGU  

19. If tan A = cot B, then find the value of (A + B). (1)


Sol. tan A = cot B
⇒ tan A = tan (90° – B)
⇒ A = 90° – B
⇒ (A + B) = 90°
20. Find the class marks of the classes 15 – 35 and 45 – 60. (1)
Sol. Class mark of the class 15–35
.QYGT ENCUU NKOKV 7RRGT ENCUU NKOKV
=

 +  
= = = 25
 
Class mark of the class 45–60
.QYGT ENCUU NKOKV 7RRGT ENCUU NKOKV
=

 +  
= = = 52.5
 

SECTION–B
Q. Nos. 21 to 26 carry 2 marks each.
21. A teacher asked 10 of his students to write a polynomial in one variable on a paper and then to
handover the paper. The following were the answers given by the students: (2)

 
2x + 3, 3x2 + 7x + 2, 4x3 + 3x2 + 2, x3 + Z + 7, 7x +  , 5x3 – 7x + 2, 2x2 + 3 – , Z − ,
Z 

ax3 + bx2 + cx + d, Z +
Z
Answer the following questions
(i) How many of the above ten, are not polynomials?
(ii) How many of the above ten, are quadratic polynomials?
Sol. (i) The answers that are not polynomials are as follows:

Z +  Z +  2x2 + 3 –  , Z + 
Z Z
(ii) Only one i.e., 3x2 + 7x + 2 of the above ten is a quadratic polynomial.
22. Compute the mode for the following frequency distribution: (2)

Size of items 0–4 4–8 8–12 12–16 16–20 20–24 24–28


(in cm)
Frequency 5 7 9 17 12 10 6

Click Here to Buy Main Book Online


M-26 CBSE BOARD EXAMINATION PAPER–2020 (SOLVED)
Sol. Here, the maximum frequency is 17 and the class corresponding to this frequency is 12–16. So, the modal
class is 12–16. Therefore,
l = 12; h = 4
f1 = 17; f0 = 9; f2 = 12
⎛ H − H ⎞ ⎛  −  ⎞
∴ Mode = N + ⎜ ⎟ × J =  + ⎜ ⎟×

⎝  H − H − H  ⎠ ⎝  ×  −  −  ⎠

=  + = 12 + 2.46 = 14.46 cm

23. In Fig., ABC and DBC are two triangles on the same base BC. If AD intersects BC at O, show that
CT
Δ #$% #1
 . (2)
CT
Δ &$% &1

C
O

OR
In Fig., if AD ⊥ BC, then prove that AB2 + CD2 = BD2 + AC2.
C

B
A

Sol. Given: ABC and DBC are two triangles on the same base BC. AD intersects BC at O.

CT
Δ #$% #1
To prove: =
CT
Δ &$% &1
A
Construction: Draw AM ⊥ BC and DN ⊥ BC
Proof: In Δ AOM and Δ DON C
∠ AMO = ∠ DNO | Each = 90° (By Construction) O
N

∠ AOM = ∠ DON | Vertically opposite angles M


∴ Δ AOM ~ Δ DON | AA similarity criterion B

#/ #1
∴ = …(1) D
&0 &1
|Q Corresponding sides of two similar triangles are proportional

Click Here to Buy Main Book Online


MATHEMATICS–X M-27

CT
Δ #$% |
$% |
#/

Now =  Q Area of a triangle = × Base × Altitude
CT
Δ &$%  
|
$% |
&0

#/ #1
=  |From (1)
&0 &1
OR
Given: AD ⊥ BC
To prove: AB2 + CD2 = BD2 + AC2
Proof: In right triangle ADB,
AB2 = AD2 + BD2 | By Pythagoras Theorem
⇒ AB2 – BD2 = AD2 …(1)
In right triangle ADC,
AC2 = AD2 + CD2 | By Pythagoras Theorem
⇒ AC – CD2 = AD2
2 …(2)
From (1) and (2), equating the two values of AD2, we get
AB2 – BD2 = AC2 – CD2
⇒ AB2 + CD2 = BD2 + AC2
EQV  D
24. Prove that  = cosec a (2)
 EQUGE D
OR
Show that tan4 q + tan2 q = sec4 q – sec2 q

EQV α
Sol. LHS =  +
 + EQUGE α

EQUGE  α − 
= + |Q cosec2 α – cot2 α = 1
 + EQUGE α


EQUGE α − 
EQUGE α + 
= +
EQUGE α + 
= 1 + (cosec α – 1) = cosec α = RHS
OR
RHS = sec4 θ – sec2 θ
= sec2 θ (sec2 θ – 1)
= (1 + tan2 θ) (1 + tan2 θ – 1) |Q sec2 θ – tan2 θ = 1
= (1 + tan2 θ) tan2 θ
= tan2 θ + tan4 θ = tan4 θ + tan2 θ = LHS
25. A child has a die whose six faces show the letters as shown below: (2)

# # $ % % %

The die is thrown once. What is the probability of getting (i) A, (ii) C?
Sol. Total number of letters = 6
∴ Number of all possible outcomes = 6

Click Here to Buy Main Book Online


M-28 CBSE BOARD EXAMINATION PAPER–2020 (SOLVED)
(i) Let E1 be the event of getting A. Then number of outcomes favourable to E1 = 2 |Q There are 2 A’s
0WODGT QH QWVEQOGU HCXQWTCDNG VQ '  
∴ P(E1) = = =
0WODGT QH CNN RQUUKDNG QWVEQOGU  
(ii) Let E2 be the event of getting C. Then number of outcomes favourable to E2 = 3 |Q There are 3 C’s
0WODGT QH QWVEQOGU HCXQWTCDNG VQ '  
∴ P(E2) = = =
0WODGT QH CNN RQUUKDNG QWVEQOGU  
26. A solid is in the shape of a cone mounted on a hemisphere of same base radius. If the curved
surface areas of the hemispherical part and the conical part are equal, then find the ratio of the
radius and the height of the conical part. (2)
Sol. Let the radius of the base, height and the slant height of the conical part be r, h and l
respectively. Then, radius of the hemispherical part = r
Also, l2 = r2 + h2 …(1)
l
According to the question, h

Curved surface area of the hemi-spherical part


r
= Curved surface area of the conical part
⇒ 2πr2 = πrl
⇒ 2r = l
⇒ 2r = T  + J | From (1)
⇒ 4r2 = r2 + h2 | Squaring both sides

T 
⇒ 3r2 = h2 ⇒ 
=
J 
T 
⇒ = ⇒ r : h =  
J 
Hence, the ratio of the radius and the height of the conical part is 1 : 

SECTION–C
Question no 27 to 34 carry 3 marks each.
27. In Fig., if D ABC ~ D DEF and their sides of lengths (in cm) are marked along them, then find the
lengths of sides of each triangle. (3)
D

18 6x
2x – 1 3x

B C E F
2x + 2 3x + 9

Sol. Q Δ ABC ~ Δ DEF | Given

#$ $% %#
∴ = 
&' '( (&
| Q Corresponding sides of two similar triangles are proportional
Z −  Z +  Z Z −  Z +  
⇒ = = ⇒ = =
 Z +   Z  Z +  

Click Here to Buy Main Book Online


MATHEMATICS–X M-29
From first and third ratios,
Z −   Z − 
= ⇒ =1
  
⇒ 2x – 1 = 9 ⇒ 2x = 9 + 1
⇒ 2x = 10
⇒ x=5 …(1)
From second and third ratios,
Z +  
=
Z +  
⇒ 2(2x + 2) = 3x + 9 | Cross-multiplying
⇒ 4x + 4 = 3x + 9
⇒ 4x – 3x = 9 – 4
⇒ x=5 …(2)
From (1) and (2), we get
x=5
For D ABC
AB = 2x – 1 = 2(5) – 1 = 9 cm
BC = 2x + 2 = 2(5) + 2 = 12 cm
CA = 3x = 3(5) = 15 cm
Hence, the lengths of the sides AB, BC and CA of Δ ABC are 9 cm, 12 cm and 15 cm respectively.
For D DEF
DE = 18 cm
EF = 3x + 9 = 3(5) + 9 = 24 cm
FD = 6x = 6(5) = 30 cm
Hence, the lengths of the sides DE, EF and FD of Δ DEF are 18 cm, 24 cm and 30 cm respectively.
28. If a circle touches the side BC of a triangle ABC at P and extended sides AB and AC at Q and R,
respectively, prove that (3)


AQ =
$% %# #$

Sol. Given: A circle touches the side BC of a triangle ABC at P and extended sides AB and AC at Q and R,
respecitvely.


To Prove: AQ =
$% %# #$

Proof: AQ = AB + BQ A
= AB + BP …(1)
Q Tangent segments from an external
point to a circle are equal in length B P C
AQ = AR Q Tangent segments from an external R
point to a circle are equal in length Q
= AC + CP …(2)
Adding (1) and (2), we get
2AQ = (AB + BP) + (AC + CP)
= (BP + CP) + CA + AB
= BC + CA + AB

⇒ AQ =
$% %# #$


Click Here to Buy Main Book Online


M-30 CBSE BOARD EXAMINATION PAPER–2020 (SOLVED)
29. The area of a circular play ground is 22173 m2. Find the cost of fencing this ground at the rate of
` 50 per metre. (3)
Sol. Let the radius of circular playground be r m . Then area of the circular playground
= r2 m2 …(1)
According to the question,
Area of the circular playground =22176 m2 …(2)
From (1) and (2)
r2 = 22176

22 2 r2
 r = 22176  = 1008
7 7
 r2 = 1008 × 7  r2 = 7056
 r= 7056  r = 84 m
2 7056
2 3528
2 1764
2 882
3 441
3 147
7 49
7 7
1
 7056 = 2 × 2 × 2 × 2 × 3 × 3 × 7 × 7
 7056 = 2 × 2 × 3 × 7 = 84
 Circumference of the playground
22
= 2r = 2 × × 84 = 528 m
7
 Cost of fencing the ground at the rate of ` 50 per metre
= 528 × 50 = ` 26400

y 
30. If 2x + y = 23 and 4x – y = 19, find the value of (5y – 2x) and   2  . (3)
x 
OR

1 1 11
Solve for x:  = , x = – 4, 7.
x + 4 x  7 30
Sol. We have,
2x + y = 23 …(1)
4x – y = 19 …(2)
Adding equations (1) and (2), we get
42
6x = 42  x= =7
6
Putting this value of x in equation (1), we get
2(7) + y = 23  14 + y = 23
 y = 23 – 14 = 9
 5y – 2x = 5(9) – 2(7) = 45 – 14 = 31
y 9 9  14  5
and 2 = 2 = 
x 7 7 7

Click Here to Buy Main Book Online


MATHEMATICS–X M-31
OR

We have,
  
− = , x ≠ – 4, 7
Z+ Z− 

Z −  −
Z +   −  
⇒ = ⇒ =

Z + 
Z −   Z
Z −  + 
Z −  
−   −  
⇒ 
= ⇒ 
=
Z −  Z +  Z −   Z −  Z −  
⇒ 2
x – 3x – 28 = – 30 ⇒ x2 – 3x + 2 = 0
⇒ 2
x – x – 2x + 2 = 0 ⇒ x(x – 1) – 2(x – 1) = 0
⇒ (x – 1) (x – 2) = 0
⇒ x – 1 = 0 or x–2=0
⇒ x = 1 or x=2
⇒ x = 1, 2
Hence, the required roots are 1 and 2.
31. If the mid-point of the line segment joining the points A(3, 4) and B(k, 6) is P(x, y) and x + y – 10 = 0,
find the value of k. (3)
OR
Find the area of triangle ABC with A(1, – 4) and the mid-points of sides through A being (2, – 1)
and (0, – 1).
Sol. A B
(3, 4) P (k, 6)
(x, y)
Q P(x, y) is the mid-point of the line segment joining the ponts A(3, 4) and B(k, 6)
+M
∴ x=

+
y= =5

Q x + y – 10 = 0
+M +M
∴ +  −  = 0 ⇒ − =0
 
+M
⇒ =5 ⇒ 3 + k = 10

⇒ k=7
OR
Let B → (h, k)
Q P(2, – 1) is the mid-point of side AB
+J A (1, –4)
∴ 2=


−+M
–1= (2, –1) P Q (0, –1)

⇒ h=3
k=2
B C
∴ B → (3, 2)

Click Here to Buy Main Book Online


M-32 CBSE BOARD EXAMINATION PAPER–2020 (SOLVED)
Let C → (l, m)
Q Q(0, – 1) is the mid-point of side AC
+N
∴ 0=

−+O
–1=

⇒ l=–1
m=1
∴ C → (– 1, 1)
Now, area of triangle ABC

= [1(2 – 1) + 3(1 – (– 4)) + (– 1) (– 4 – 2)]

 
= [1 + 15 + 6] =
 
= 11 square units
32. If in an A.P. the sum of first m terms is n and the sum of its first n terms is m, then prove that the
sum of its first (m + n) terms is – (m + n). (3)
OR
Find the sum of all 11 terms of an A.P. whose middle term is 30.
Sol. Let a be the first term and d be the common difference of the A.P. Then,
Sm = n
O
⇒ =C +
O −  F ? = n

⇒ 2am + m(n – 1)d = 2n …(1)
and Sn = m
P
⇒ =C +
P −  F ? = m

⇒ 2an + n(n – 1)d = 2m …(2)
Subtracting equation (2) from equation (1), we get
2a(m – n) + [m(m – 1) – n(n – 1)]d = 2(n – m)
⇒ 2a(m – n) + [m2 – m – n2 + n]d = – 2(m – n)
⇒ 2a(m – n) + [(m2 – n2) – (m – n)]d = – 2(m – n)
⇒ 2a(m – n) + [(m – n) (m + n) – (m – n)]d = – 2(m – n)
⇒ 2a(m – n) + (m – n) (m + n – 1)d = – 2(m – n)
⇒ 2a + (m + n – 1)d = – 2 …(3)
| Dividing throughout by (m – n)
O+P
Now, Sm+n = =C +
O + P −  F ?

O+P
=
−  | From (3)

= – (m + n)
OR
Let a be the first term and d be the common difference of the A.P.
Number of all terms of AP = 11 which is odd.

Click Here to Buy Main Book Online


MATHEMATICS–X M-33
VJ
⎛   ⎞
∴ Middle term = ⎜ ⎟ VGTO = 6th term
⎝  ⎠
= a + (6 – 1)d |Q an = a + (n – 1)d
= a + 5d
According to the question,
a + 5d = 30 …(1)
Now, sum of all 11 terms
 P
= =C +
 −  F ? ' 5P = = C +
P −  F ?
 

= =C +  F ? = 11(a + 5d)

= 11 × 30 | From (1)
= 330
33. A fast train takes 3 hours less than a slow train for a journey of 600 km. If the speed of the slow
train is 10 km/h less than that of the fast train, find the speed of each train. (3)
Sol. Let the speed of the fast train be x km/h
Then, speed of the slow train = (x – 10) km/h
 &KUVCPEG
Time taken by the fast train for a journey of 600 km = J ' 6KOG 
Z 5RGGF
 &KUVCPEG
Time taken by the slow train for a journey of 600 km = J ' 6KOG 
Z −  5RGGF
According to the question,

 
− =3
Z −  Z

⎛  ⎞ ⎧ Z −
Z −  ⎫
⇒  ⎜ − ⎟ =3 ⇒  ⎨ ⎬ =1
⎝ Z −  Z⎠ ⎩
Z −  Z ⎭


⇒ 
=1 ⇒ x2 – 10x = 2000
Z −  Z

⇒ x2 – 10x – 2000 = 0 ⇒ x2 – 50x + 40x – 2000 = 0


⇒ x(x – 50) + 40(x – 50) = 0 ⇒ (x – 50) (x + 40) = 0
⇒ x – 50 = 0 or x + 40 = 0
⇒ x = 50 or x = – 40
x = – 40 is inadmissible as x is the speed of the train which cannot be negative.
∴ x = 50 ⇒ x – 10 = 40
Hence, the speeds of the fast train and the slow train are 50 km/h and 40 km/h respectively.


34. If 1 + sin2 q = 3 sin q cos q, prove that tan q = 1 or (3)

Sol. We have
1 + sin2 θ = 3 sin θ cos θ

Click Here to Buy Main Book Online


M-34 CBSE BOARD EXAMINATION PAPER–2020 (SOLVED)
 + UKP  θ  UKP θ EQU θ
⇒ = |dividing both sides by cos2 θ
EQU θ EQU θ

 UKP  θ  UKP θ
⇒ + =

EQU θ EQU θ EQU θ
⇒ sec2 θ + tan2 θ = 3 tan θ
⇒ (1 + tan2 θ) + tan2 θ = 3 tan θ |Q sec2 θ – tan2 θ = 1
⇒ 1 + 2 tan2 θ = 3 tan θ
⇒ 2 tan2 θ – 3 tan θ + 1 = 0
⇒ 2
2 tan θ – 2 tan θ – tan θ + 1 = 0
⇒ 2 tan θ (tan θ – 1) – 1(tan θ – 1) = 0
⇒ (tan θ – 1) (2 tan θ – 1) = 0
⇒ tan θ – 1 = 0 or 2 tan θ – 1 = 0
⇒ tan θ = 1 or 2 tan θ = 1

⇒ tan θ = 1 or tan θ =


⇒ tan θ = 1 or


SECTION–D
Question Nos. 35 to 40 carry 4 marks each
35. The mean of the following frequency distribution is 18. The frequency f in the class interval 19–21
is missing. Determine f. (4)

Class Interval 11–13 13–15 15–17 17–19 19–21 21–23 23–25


Frequency 3 6 9 13 f 5 4

OR
The following table gives production yield per hectare of wheat of 100 farms of a village

Production yield 40–45 45–50 50–55 55–60 60–65 65–70


Frequency 4 6 16 20 30 24

Change the distribution to a ‘more than’ type distribution and draw its ogive.

Sol. Class Interval Class Mark Frequency fixi


xi fi

11–13 12 3 36
13–15 14 6 84
15–17 16 9 144
17–19 18 13 234
19–21 20 f 20f
21–23 22 5 110
23–25 24 4 96

Total f + 40 20f + 704

Click Here to Buy Main Book Online


MATHEMATICS–X M-35
ΣHK ZK
Z =
ΣH K
 H + 
⇒ 18 = ⇒ 18(f + 40) = 20f + 704
H + 
⇒ 18f + 720 = 20f + 704 ⇒ 20f – 18f = 720 – 704
⇒ 2f = 16 ⇒ f=8
OR
More than type distribution
Production yield No. of farms
More than 40 100
More than 45 96
More than 50 90
More than 55 74
More than 60 54
More than 65 24

We mark the lower limits of the class intervals on the horizontal axis (x-axis) and their corresponding
cumulative frequencies on the vertical axis (y-axis). Then, we plot the point (40, 100), (45, 96), (50, 90),
(55, 74), (60, 54) and (65, 24) on the graph paper and join them by a free hand smooth curve.
The curve we get is a ‘more than type’ ogive.

(40, 100) ‘More than Type’ ogive


100 (45, 96)
(50, 90)
90

80 (55, 74)
Cumulative frequency

70

60 (60, 54)

50

40

30 (65, 24)

20

10

X
O 40 45 50 55 60 65

Lower limits

Q
36. Find the area of the shaded region in Fig., if PQ = 24 cm, PR = 7 cm and O is the
centre of the circle. (4)
OR O

Find the curved surface area of the frustum of a cone, the diameters of whose
circular ends are 20 m and 6 m and its height is 24 m.
R P

Click Here to Buy Main Book Online


M-36 CBSE BOARD EXAMINATION PAPER–2020 (SOLVED)
Sol. ∠ RPQ = 90° | Angle in a semi-circle is 90°
∴ RQ2 = PR2 + PQ2 | By Pythagoras Theorem
= 72 + 242 = 49 + 576 = 625
⇒ RQ =  = 25 cm

∴ Radius of the circle (r) = EO


1. Area of the semi-circle = πT 


 ⎛  ⎞ 
= π⎜ ⎟ = π
 ⎝  ⎠ 
   ×  
= × = = EO 
  × 
 
2. Area of right-triangle RPQ = ª 23 ª 24 Area of a triangle = × Base × Altitude
 

×  ×  = 84 cm2
=

∴ Area of shaded region = Area of the semi-cirlce RPQR – Area of right triangle RPQ

= (1) – (2) = − 

 −  
= = EO 
 

Hence, the area of the shaded region is EO .

OR

Here, r1 = = 10 m


r2 = =3m

h = 24 m
l2 = h2 + (r1 – r2)2 = (24)2 + (10 – 3)2
= (24)2 + (7)2 = 576 + 49 = 625
∴ l =  = 25 m
∴ Curved surface area = πl(r1 + r2)
= π · 25 · (10 + 3) = π · 25 · 13 = 325 π
  
=  × = O
 
37. Prove that  is an irrational number. (4)
Sol. Let us assume, to the contrary, that  is a rational number.
Then, we can find co-prime integers a and b (≠ 0) such that

C
 = ⇒ D = a
D
Squaring both sides, we get
5b2 = a2
Therefore, 5 divides a2.

Click Here to Buy Main Book Online


MATHEMATICS–X M-37
Therefore, 5 divides a.
| Let p be a prime number. If p divides a2, then p divides a where a is a positive integer
So, we can write
a = 5c for some integer c
Substituting for a, we get
5b2 = 25c2 ⇒ b2 = 5c2
Therefore, 5 divides b2
Therefore, 5 divides b
| Let p be a prime number. If p divides a2, then p divides a where a is a positive integer
Therefore, a and b have at least 5 as a common factor.
But this contradicts the fact that a and b have no common factor other than 1.

This contradiction has arisen because of our incorrect assumption that  is a rational number.

So, we conclude that  is an irrational number.


38. It can take 12 hours to fill a swimming pool using two pipes. If the pipe of larger diameter is used
for four hours and the pipe of smaller diameter for 9 hours, only half of the pool can be filled.
How long would it take for each pipe to fill the pool separately? (4)
Sol. Let the pipe of larger diameter take x hours to fill the swimming pool and the pipe of smaller diameter take

y hours to fill the swimming pool. Then swimming pool filled by the pipe of larger diameter in 1 hour =
Z

∴ Swimming pool filled by the pipe of larger diameter in 12 hours =
Z

Swimming pool filled by the pipe of smaller diameter in 1 hour =
[


∴ Swimming pool filled by the pipe of smaller diameter in 12 hours =
[
According to the first conditon of the problem,
  ⎛ ⎞
+ =1 ⇒  ⎜ + ⎟ = 1
Z [ ⎝ Z [⎠
  
⇒ + = …(1)
Z [ 

Again, swimming pool filled by the pipe of larger diameter in 4 hours =
Z

Swimming pool filled by the pipe of smaller diameter in 9 hours =
[
According to the second condition of the problem,
  
+ = …(2)
Z [ 
 
Put = X and = Y. Then, equations (1) and (2) respectively reduce to
Z [

X+Y= …(3)


4X + aY = …(4)


Click Here to Buy Main Book Online


M-38 CBSE BOARD EXAMINATION PAPER–2020 (SOLVED)
Multiplying equations (3) by 4, we get

4X + 4Y = …(5)

Subtracting equation (5) from equation (4), we get
  −
5Y = − ⇒ 5Y =
  
 
⇒ 5Y = ⇒ Y=
 
  
⇒ = ' =;
[  [
⇒ y = 30

Substituting Y = in equation (3), we get

 
X+ =
 
  −  
⇒ X= − = = =
    
  
⇒ = ' =:
Z  Z
⇒ x = 20
Hence, the times taken by the pipe of larger diameter and the pipe of smaller diameter to fill the pool
separately are 20 hours and 30 hours respectively.
39. Draw two tangents to a circle of radius 4 cm, which are inclined to each other at an angle of 60°.
(4)
OR
Construct a triangle ABC with sides 3 cm, 4 cm and 5 cm. Now, construct another triangle whose

sides are times the corresponding sides of Δ ABC.

Sol. Required: To draw two tangents to a circle of radius 4 cm, which are inclined to each other at an angle
of 60°. A
Steps of Construction:
m

1. Draw a circle of radius 4 cm with centre O.


5c

2. Draw an angle AOB of 120° O 120° C


3. At A and B, draw 90° angles which meet at C.
5c

Then, AC and BC are the required tangents which are inclined to each
m

other at an angle of 60°. B


OR
Required: To construct a triangle ABC with sides 3 cm, 4 cm and 5 cm and then to construct another

triangle whose sides are times the corresponding sides of Δ ABC.

Steps of Construction:
1. Draw a triangle ABC with AB = 3 cm, BC = 4 cm and CA = 5 cm
2. Draw any ray BX making an acute angle with BC on the side opposite to the vertex A
3. Locate 5 points B1, B2, B3, B4 and B5 on BC such that
BB1 = B1B2 = B2B3 = B3B4 = B4B5

Click Here to Buy Main Book Online


MATHEMATICS–X M-39
4. Join B5C. Draw a line through B4 parallel to the line B5C intersecting BC at C′.
A


5 cm
3 cm

B C
4 cm C¢

B1

B2

B3

B4

B5

5. Draw a line through C′ parallel to the line CA to intersect BA at A′.


Then, Δ A′BC′ is the required triangle.
40. The angle of elevation of the top of a building from the foot of a tower is 30° and the angle of
elevation of the top of a tower from the foot of the building is 60°. If the tower is 50 m high, then
find the height of the building. (4)
Sol. Let the height of the building be h m. Then,
AB = h m …(1) P (Top)
In right triangle PQB
23 
tan 60° = ⇒  =
$3 $3

⇒ BQ = O …(2)

Tower

In right triangle ABQ, (Top)
A
#$
tan 30° =
Building
hm

$3

 J 60° 30°
⇒ = | From (1) and (2) B Q
 ⎛  ⎞ (Foot) (Foot)
⎜ ⎟
⎝ ⎠

 J
⇒ = ⇒ 3h = 50
 

 
⇒ h= =  O
 


Hence, the height of the building is  O.


Click Here to Buy Main Book Online


MRP: 575.00

Click Here to Buy the Book Online

Committed to Educate the Nation NEW AGE INTERNATIONAL (P) LIMITED, PUBLISHERS
LONDON • NEW DELHI • NAIROBI

IN INDIA Visit us at www.newagegolden.com


M-40 CBSE BOARD EXAMINATION PAPER–2019

Series : JMS/5 SET-1


Code No. 30/5/1

CBSE BOARD EXAMINATION PAPER-2019


Mathematics–X (Solved)

Time Allowed: 3 hours Maximum Marks: 80


General Instructions:
(i) All questions are compulsory.
(ii) The question paper consists of 30 questions divided into four sections — A, B, C and D.
(iii) Section A contains 6 questions of 1 mark each. Section B contains 6 questions of 2 marks each, Section C
contains 10 questions of 3 marks each and Section D contains 8 questions of 4 marks each.
(iv) There is no overall choice. However, an internal choice has been provided in two questions of 1 mark each,
two questions of 2 marks each, four questions of 3 marks each and three questions of 4 marks each. You
have to attempt only one of the alternatives in all such questions.
(v) Use of calculators is not permitted.

SECTION–A
1. The HCF of two numbers a and b is 5 and their LCM is 200. Find the product ab.
Sol. Product ab = HCF (a, b) × LCM (a, b)
= 5 × 200 = 1000
2. Find the value of k for which x = 2 is a solution of the equation kx2 + 2x – 3 = 0.
OR
Find the value/s of k for which the quadratic equation 3x2 + kx + 3 = 0 has real and equal roots.
Sol. If x= 2 is a solution of the equation
kx2 + 2x – 3 = 0, then
k(2)2 + 2(2) – 3 = 0
 4k + 4 – 3 = 0  4k + 1 = 0
1
 4k = –1  k=–
4

OR
The given quadratic equation is
3x2 + kx + 3 = 0
Here, a = 3, b = k, c = 3
 Discriminant (D) = b2 – 4ac = (k)2 – 4(3) (3)
= k2 – 36
For real and equal roots,
D=0
 k2 – 36 = 0  k2 = 36  k=6

M-40

Click Here to Buy Main Book Online


MATHEMATICS–X M-41
3. If in an A. P., a = 15, d = – 3 and an = 0, then find the value of n.
Sol. Here, a = 15, d = – 3, an = 0
We know that
an = a + (n – 1) d
 0 = 15 + (n – 1) (– 3)
15
 3(n – 1) = 15  n–1= |dividing by 3
3
 n–1=5  n=5+1  n=6
4. If sin x + cos y = 1; x = 30 and y is an acute angle, find the value of y.

OR
Find the value of (cos 48 – sin 42).
Sol. If sin x + cos y = 1; x = 30 and y is an acute angle, find the value of y.
 sin 30° + cos y = 1 | x = 30° (given)
1
 + cos y = 1
2
1 1
 cos y = 1 – =
2 2
 cos y = cos 60° | y is an acute angle
 y = 60°

OR
cos 48° – sin 42°
= cos (90° – 42°) – sin 42°
= sin 42° – sin 42° | cos (90° – ) = sin 
=0
5. The area of two similar triangles are 25 sq. cm and 121 sq. cm. Find the ratio of their corresponding
sides.
Sol. We know that the ratio of the areas of two similar triangles is equal to the square of the ratio of their
corresponding sides.
 Square of the ratio of their corresponding sides
2
25  5 
= =  
121  11 
5
 Ratio of their corresponding sides = = 5 : 11
11
6. Find the value of ‘a’ so that the point (3, a) lies on the line represented by 2x – 3y = 5.
Sol. If the point (3, a) lies on the line represented by 2x – 3y = 5, then
2(3) – 3(a) = 5
 6 – 3a = 5  3a = 6 – 5
1
 3a = 1  a=
3

Click Here to Buy Main Book Online


M-42 CBSE BOARD EXAMINATION PAPER–2019
SECTION–B
7. If Sn, the sum of the first n terms of an A.P. is given by Sn = 2n2 + n, then find its nth term.

OR
If the 17th term of an A.P. exceeds its 10th term by 7, find the common difference.
Sol. Sn = 2n2 + n
Put n = 1, 2
S1 = 2(1)2 + 1 = 2 + 1 = 3
S2 = 2(2)2 + 2 = 8 + 2 = 10
 a 1 = S1 = 3
a2 = S2 – S1 = 10 – 3 = 7
 a = a1 = 3
d = a2 – a1 = 7 – 3 = 4
 nth term = an
= a + (n – 1)d
= 3 + (n – 1) 4
= 3 + 4n – 4 = 4n – 1

OR
Let the first term and the common difference of the A.P. be a and d respectively.
Then,
17th term = a17
= a + (17 – 1)d | an = a + (n – 1) d
= a + 16d ...(1)
th
and 10 term = a10
= a + (10 – 1)d | an = a + (n – 1) d
= a + 9d ...(2)
According to the question,
a17 = a10 + 7
 a + 16d = (a + 9d) + 7 |From (1) and (2)
 16d = 9d + 7  16d – 9d = 7  7d = 7
 d =1 |dividing by 7
Hence, the common difference is 1.
8. The mid-point of the line segment joining A(2a, 4) and B(–2, 3b) is (1, 2a + 1). Find the values of a
and b.

Sol.

Here, A (2a, 4)
B (–2, 3b)
Let C be the mid-point of AB. Then,
C (1, 2a + 1)
 C is the mid-point of AB

Click Here to Buy Main Book Online


MATHEMATICS–X M-43
 By mid-point formula,
2a  (2)
= 1 ...(1)
2
4  3b
and = 2a + 1 ...(2)
2
From (1), 2a – 2 = 2
 2a = 2 + 2 = 4

4
 a = =2 ...(3)
2
From (2), 4 + 3b = 2 (2a + 1)
 4 + 3b = 4a + 2
 4 + 3b = 4(2) + 2 |From (3)
 4 + 3b = 8 + 2
 4 + 3b = 10  3b = 10 – 4 = 6

6
 b = =2 ...(4)
3
Hence, a = 2, b = 2
9. A child has a die whose 6 faces show the letters given below:

A B C A A B
The die is thrown once. What is the probability of getting (i) A (ii) B ?
Sol. Total number of letters = 6
 Number of all possible outcomes = 6
(i) A
Let E be the event of getting A.
Then, the number of outcomes favourable to E is 3 as there are 3 A' s.

Number of outcomes favourable to E 3 1


 P(E) = = 
Number of all possible outcomes 6 2
(ii) B
Let F be the event of getting B.
Then, the number of outcomes favourable to F is 2 as there are 2 B' s.
Number of outcomes favourable to F
 P(F) =
Number of all possible outcomes
2 1
= 
6 3
10. Find the HCF of 612 and 1314 using prime factorisation.

OR
Show that any positive odd integer is of the form 6m + 1 or 6m + 3 or 6m + 5, where m is some
integer.

Click Here to Buy Main Book Online


M-44 CBSE BOARD EXAMINATION PAPER–2019

Sol.

612 = 2 × 2 × 3 × 3 × 17 1314 = 2 × 3 × 3 × 73
= 22 × 32 × 17 = 2 × 32 × 73
 HCF (612, 1314) = 2 × 32 = 2 × 9 = 18
OR
Let us start with taking a, where a is any positive odd integer. We apply the division algorithm with a and
b = 6.
Since 0  r < 6, the possible remainders are 0, 1, 2, 3, 4, 5. That is, a can be 6m or 6m + 1 or 6m + 2 or
6m + 3 or 6m + 4 or 6m + 5, where m is the quotient.
However, since a is odd, we do not consider the cases 6m, 6m + 2 or 6m + 4 (since all the three are even
being divisible by 2).
Therefore, any positive odd integer is of the form 6m + 1 or 6m + 3 or 6m + 5, where m is some integer.
11. Cards marked with numbers 5 to 50 (one number on one card) are placed in a box and mixed
thoroughly. One card is drawn at random from the box. Find the probability that the number on
the card taken out is (i) a prime number less than 10, (ii) a number which is a perfect square.
Sol. Total number of cards = 46
 Number of all possible outcomes = 46
(i) a prime number less than 10
Prime numbers less than 10 are 5, 7.
Their number = 2
 Number of favourable outcomes = 2
 P (a prime number less than 10)
Number of favourable outcomes 2 1
= =
Number of all possible outcomes 46 23
(ii) a number which is a perfect square
Numbers which are perfect square are 9, 16, 25, 36, 49
Their number = 5
 Number of favourable outcomes = 5
 P (a number which is a perfect square)
Number of favourable outcomes 5
= =
Number of all possible outcomes 46
12. For what value of k, does the system of linear equations
2x + 3y = 7
(k – 1) x + (k + 2) y = 3k
have an infinite number of solutions ?

Click Here to Buy Main Book Online


MATHEMATICS–X M-45
Sol. The given system of linear equations is
2x + 3y = 7
(k – 1)x + (k + 2)y = 3k
 2x + 3y – 7 = 0
(k – 1)x + (k + 2)y – 3k = 0
Here, a1 = 2, b1 = 3, c1 = –7
a2 = k – 1, b2 = k + 2, c2 = – 3k
For infinite number of solutions,
a1 b c
= 1 = 1
a2 b2 c2
2 3 7 2 3 7
 = =  = =
k1 k2 3k k1 k2 3k
2 3
From first two, =
k1 k2
 2(k + 2) = 3 (k – 1)  2k + 4 = 3k – 3
 3k – 2k = 4 + 3  k=7
3 7
From last two, =
k2 3k
 3(3k) = 7(k + 2)  9k = 7k + 14
 9k – 7k = 14  2k = 14
14
 k= =7
2
Hence, the required value of k is 7.

SECTION–C
13. Prove that 5 is an irrational number.
Sol. Let us assume, to the contrary, that 5 is a rational number.
So, we can find co-prime integers a and b ( 0) such that
a
5 =  5 b=a
b
Squaring both sides, we get
5b2 = a2
2
Therefore, 5 divides a .
Therefore, 5 divides a.
Let p be a prime number. Let p divide a2. Then,
p divides a, where a is a positive integer.
So, we can write
a = 5c for some integer c.
Substituting for a, we get
5b2 = 25c2
 b2 = 5c2
This means that 5 divides b2.
Therefore, 5 divides b.
Let p be a prime number. Let p divide a2. Then,
p divides a, where a is a positive integer.

Click Here to Buy Main Book Online


M-46 CBSE BOARD EXAMINATION PAPER–2019
Therefore, a and b have at least 5 as a common factor.
But this contradicts the fact that a and b have no common factor other than 1.
This contradiction arose because of our incorrect assumption that 5 is a rational number.
So, we conclude that 5 is an irrational number.

14. Find all the zeroes of the polynomial x4 + x3 – 14x2 – 2x + 24, if two of its zeroes are 2 and – 2 .
Sol. Let p (x) = x4 + x3 – 14x2 – 2x + 24
Since two of zeroes of p (x) are 2 and – 2 , therefore,
(x – 2 ) {x – (– 2 )}
i.e., (x – 2 ) (x + 2 )
i.e., x2 – 2
is a factor of p (x).
Let us divide p (x) by x2 – 2. Then, we get
p(x) = (x2 – 2) (x2 – x – 12)
2
x + x – 12
4 3 2
2 x + x – 14x – 2x + 24
x –2 4 2
x – 2x
– +
3 2
x – 12x – 2x + 24
3
x – 2x
– +
2
– 12x + 24
2
– 12x + 24
+ –
0

 The other two zeroes of p(x) are given by


x2 – x – 12 = 0
 x2 – 4x + 3x – 12 = 0  x (x – 4) + 3 (x – 4) = 0
 (x – 4) (x + 3) = 0  x– 4 = 0 or x + 3 = 0
 x = 4 or x = – 3  x = 4, – 3
Hence, all the zeroes of the given polynomial are 2 , – 2 , 4 and –3.

AP 1
15. Point P divides the line segment joining the points A(2, 1) and B(5, – 8) such that = . If P lies
AB 3
on the line 2x – y + k = 0, find the value of k.

OR
For what value of p, are the points (2, 1), (p, – 1) and (–1, 3) collinear ?

Sol.

AP 1
=
AB 3

Click Here to Buy Main Book Online


MATHEMATICS–X M-47
 3 AP = AB  3 AP = AP + PB
 3 AP – AP = 2PB
AP 1
 2 AP = PB  =
PB 2
 P divides AB in the ratio 1 : 2 internally.
 By section formula,
 (1) (5)  (2) (2) (1) (8)  (2) (1) 
P  , 
 12 12 

 P  
5  4 8  2
3
,
3 
 P   
9 6
3 3
,

 P  {3, –2}
If P (3, –2) lies on the line 2x – y + k = 0,
Then, 2(3) – (–2) + k = 0
 6+2+k=0
 8 + k = 0  k = –8
Hence, the required value of k is –8.

OR
Let A  (2, 1), B (p, –1) and C (–1, 3). If the points A, B and C are collinear, then area of ABC = 0
1
 [ 2 (–1 – 3) + p(3 – 1) + (–1) {1 – (–1)}] = 0
2
 2 (–4) + 2p + (–1) (2) = 0  – 8 + 2p – 2 = 0
 2p – 10 = 0  2p = 10
10
 p= =5
2
Hence, the required value of p is 5.

tan  cot  cos  + sin 


16. Prove that: – =
1 – tan  1 – cot  cos  – sin 

OR

If cos  + sin  = 2 cos , show that cos  – sin  = 2 sin .


tan cot
Sol. LHS = –
1 tan 1 cot
sin  cos 
= cos   sin 
sin  cos 
1 1
cos  sin 
sin  cos 
= 
cos   sin  sin   cos 
sin  cos  sin   cos 
=  =
cos   sin  cos   sin  cos   sin 

Click Here to Buy Main Book Online


M-48 CBSE BOARD EXAMINATION PAPER–2019
cos   sin 
= cos   sin  = RHS

OR
cos  + sin  = 2 cos 
 ( 2 – 1) cos  = sin 
 ( 2 + 1) ( 2 – 1) cos  = ( 2 + 1) sin 
|Multiplying both sides by ( 2 + 1)

 {( 2 )2 – (1)2} cos  = 2 sin  + sin 


 (2 – 1) cos  = 2 sin  + sin 
 cos  = 2 sin  + sin 
 cos  – sin  = 2 sin 
17. A part of monthly hostel charges in a college hostel are fixed and the remaining depends on the
number of days one has taken food in the mess. When a student A takes food for 25 days. he has to
pay ` 4,500, whereas a student B who takes food for 30 days, has to pay ` 5,200. Find the fixed
charges per month and the cost of food per day.
Sol. Let the fixed charges per month be ` x and the cost of food per day be ` y.
Then, according to the question,
x + 25y = 4500 ...(1)
x + 30y = 5200 ...(2)
Substracting equation (1) from equation (2), we get
5y = 700
700
 y= = 140
5
Substituting this value of y in equation (2), we get
x + (30)(140) = 5200
 x + 4200 = 5200
 x = 5200 – 4200 = 1000
Hence the fixed charges per month is `1000 and the cost of food per day is ! 140.
18. In  ABC,  B = 90 and D is the mid-point of BC. Prove that AC2 = AD2 + 3CD2.
OR
In Figure, E is a point on CB produced of an isosceles  ABC, with side AB = AC. If AD  BC and
EF  AC, prove that  ABD   ECF.

Click Here to Buy Main Book Online


MATHEMATICS–X M-49
Sol. Given : In ABC, B = 90° and D is the mid-point of BC.
To Prove : AC2 = AD2 + 3CD2
Proof : In right triangle ABC,

AC2 = AB2 +BC2 ...(1) |By Pythagoras Theorem


In right triangle ABD,
AD2 = AB2 + BD2 ...(2) |By Pythagoras Theorem
Subtracting equation (2) from equation (1), we get
AC2 – AD2 = BC2 – BD2
= (2CD)2 – (CD)
1
| D is the mid-point of BC.  BD = CD = BC  BC = 2CD
2
= 4CD2 – CD2 = 3CD2
 AC2 = AD2 + 3CD2

OR
Given : In figure, E is a point on CB produced of an isosceles ABC, with side AB = AC. AD  BC and
EF  AC.
To Prove : ABD  ECF

Proof : In ABD and ECF,


 AB = AC |Given
 ACB = ABC |Angles opposite to equal sides of a triangle are equal
 ABC = ACB
 ABD = ECF ...(1)
ADB = EFC ...(2) | Each equal to 90°
From (1) and (2),
ABD ECF |by AA similarity criterion

Click Here to Buy Main Book Online


M-50 CBSE BOARD EXAMINATION PAPER–2019
19. Prove that the parallelogram circumscribing a circle is a rhombus.
Sol. Given : ABCD is a parallelogram circumscribing a circle.
To Prove : ABCD is a rhombus.
Proof : The tangent segments from an external point to a circle are equal in
length
 AP = AS
BP = BQ
CQ = CR
and DR = DS
 (AP + BP) + (CR + DR) = (AS + DS) + (BQ + CQ)
 AB + CD = AD + BC
 AB + AB = AD + AD
|opposite sides of a parallelogram are equal
 2AB = 2AD
 AB = AD
But AB = CD and AD = BC |opposite sides of a parallelogram are equal
 AB = BC = CD = DA
 Parallelogram ABCD is a rhombus.
20. In Figure, three sectors of a circle of radius 7 cm, making angles of 60, 80 and 40 at the centre
are shaded. Find the area of the shaded region.

Sol. For sector making angle of 60° at the centre


r = 7 cm
 = 60°
 60
 Area = r2 = (7)2
360 360
49
=  cm2 ...(1)
6
For sector making angle of 80° at the centre
r = 7 cm
 = 80°
 80
 Area = r2 = (7)2
360 360
98
=  cm2 ...(2)
9
For sector making angle of 40° at the centre
r = 7 cm
 = 40°

Click Here to Buy Main Book Online


MATHEMATICS–X M-51
 40
 Area = r2 = (7)2
360 360
49
=  cm2 ...(3)
9
 Area of the shaded region
= Area of the sector making an angle of 60° at the centre
+ Area of the sector making an angle of 80° at the centre
+ Area of the sector making an angle of 40° at the centre
= (1) + (2) + (3)
49 8 49 49
=    = (3 + 4 + 2)
6 9 9 18
49
= 9
18
49 49 22
= = 
2 2 7
= 7 × 11 = 77 cm2
21. The following table gives the number of participants in a yoga camp:

Age (in years): 20–30 30–40 40–50 50–60 60–70


No. of Participants: 8 40 58 90 83

Sol. Here, the maximum frequency is 90 and the class corresponding to this frequency is 50 – 60. So, the modal
class is 50 – 60.
Therefore, l = 50, h = 10, f1 = 90, f0 = 58, f2 = 83
f1  f0
 Mode = l + ×h
2 f1  f0  f2
90  58
= 50 + × 10
2(90)  58  83
32  10 32  10
= 50 + = 50 +
180  141 39
320
= 50 + = 50 + 8.2 = 58.2
39
Hence, the modal age of the participants is 58.2 years.
22. A juice seller was serving his customers using glasses as shown in Figure. The inner diameter of
the cylindrical glass was 5 cm but bottom of the glass had a hemispherical raised portion which
reduced the capacity of the glass. If the height of a glass was 10 cm, find the apparent and actual
capacity of the glass. (Use  = 3.14)

Click Here to Buy Main Book Online


M-52 CBSE BOARD EXAMINATION PAPER–2019
OR
A girl empties a cylindrical bucket full of sand, of base radius 18 cm and height 32 cm on the floor to
form a conical heap of sand. If the height of this conical heap is 24 cm, then find its slant height correct
to one place of decimal.
Sol. For cylindrical glass
Inner diameter = 5 cm
5
 Inner radius (r) = cm
2
Height (h) = 10 cm
 Apparent capacity of the glass
= r2h
5 5
= 3.14 × × × 10
2 2
= 1.57 × 5 × 5 × 5 = 1.57 × 125
= 196.25 cm3
But the actual capacity of the glass is less by the volume of the hemisphere at the bottom of the glass, i.e.,
it is less by

2 3 2 5 5 5 1.57  125 196.25


r = × 3.14 × × × = =
3 3 2 2 2 6 6
= 32.71 cm 3

So, the actual capacity of the glass


= apparent capacity of the glass – volume of the hemisphere
= (196.25 – 32.71) cm3
= 163.54 cm3
OR
For cylindrical bucket
Base radius (R) = 18 cm
Height (H) = 32 cm
 Volume = R2H
= (18)2 (32) cm3
For conical heap
Height (h) = 24 cm
Let the base radius be rcm
1 2 1
Then, Volume = r h = r2(24) = 8r2 cm3
3 3
According to the question,
Volume of the conical heap
= Volume of cylindrical bucket
 8r = (18)2(32)  r2 = (18)2 (4)
2

 r2 = (18)2 (2)2  r = 18 × 2 = 36 cm
 Slant height (l) of the conical heap

= r 2  h2 = (36)2  (24)2

= (12  3)2  (12  2)2 = 12 32  22

Click Here to Buy Main Book Online


MATHEMATICS–X M-53
= 12 9  4 = 12 13
= 12 × 3.605 = 43.26 cm
= 43.3 cm |correct to one place of decimal
Hence, the slant height of the conical heap correct to one place of decimal is 43.3 cm.

SECTION–D
23. A train travels 360 km at a uniform speed. If the speed had been 5 km/hr more, it would have taken
1 hr less for the same journey. Find the speed of the train.

OR
Solve for x :
1 1 1 1
= + + ; a  b  0, x  0, x  – (a + b)
a+ b+ x a b x
Sol. Let the speed of the train be x km/hr. Then, the time taken by the train to travel 360 km at a uniform speed
of x km/hr
360 Distance
= hr  Time =
x Speed
Increased speed of the train = (x + 5) km/hr.
Then, the time taken by the train to travel 360 km at the increased speed
360 Distance
= hr  Time =
x5 Speed
According to the question,
360 360
= –1
x5 x
360 360
  =1
x x5

1 1 
 360    =1
 x x  5
1 1 1
  = |dividing by 360
x x5 360
x5x 1
 =
x ( x  5) 360
5 1
 2 =
x  5x 360
 x2 + 5x = 1800
 x2 + 5x – 1800 = 0
 2
x + 45x – 40x – 1800 = 0
 x(x + 45) – 40(x + 45) = 0
 (x + 45) (x – 40) = 0
 x + 45 = 0 or x – 40 = 0
 x = – 45 or x = 40
 x = – 45, 40
x = – 45 is inadmissible as x is the speed of the train and speed cannot be negative.

Click Here to Buy Main Book Online


M-54 CBSE BOARD EXAMINATION PAPER–2019
 x = 40
Hence, the speed of the train is 40 km/hr.
OR
1 1 1 1
=  
ab x a b x
1 1 1 1
 – = 
ab x x a b
x  ( a  b  x) b a
 (a  b  x) x = ab
 ( a  b) ab
 =
( a  b  x) x ab
1 1
 – =
(a  b  x) x ab
 (a + b + x)x = – ab
 x2 + ax + bx = – ab
 2
x + ax + bx + ab = 0
 x(x + a) + b(x + a) = 0
 (x + a) (x + b) = 0
 x+a=0 or x + b = 0
 x = – a or x=–b
 x = – a, – b
24. If the sum of the first p terms of an A. P. is q and the sum of the first q terms is p; then show that the
sum of the first (p + q) terms is { – (p + q)}.
Sol. Let the first term and the common difference of the AP be a and d respectively.
Then, according to the question,
Sp = q
p
 [2a + (p – 1) d] = q
2
 2ap + p(p – 1) d = 2q ...(1)
and Sq = p
q
 [2a + (q – 1)d] = p
2
 2aq + q(q – 1)d = 2p ...(2)
Subtracting equation (2) from equation (1), we get
2a(p – q) + {p(p – 1) – q(q – 1)}d = 2(q – p)
 2a(p – q) + {p2 – q2 – p + q}d = 2(q – p)
2a(p – q) + {(p – q) (p + q) – (p – q)}= – 2(p – q)
 2a(p – q) + (p – q) (p + q + 1) = – 2(p – q)
 2a + (p + q – 1) = – 2 |dividing both sides by (p – q) ...(3)
Now, Sum of the first (p + q) terms
= Sp + q
( p  q)
= [2a + (p + q – 1)d]
2

Click Here to Buy Main Book Online


MATHEMATICS–X M-55
( p  q)
= (– 2) | By (3)
2
= – (p + q)
25. In a triangle, if the square of one side is equal to the sum of the squares of the other two sides, then
prove that the angle opposite to the first side is a right angle.
Sol. Given: A triangle ABC in which AC2 = AB2 + BC2
To Prove: B = 90°
Construction: Construct a PQR right angled at Q such that PQ = AB and QR = BC.

Proof In PQR,
PR2 =PQ2 + QR2 |By Pythagoras Theorem as Q = 90°
 PR2 =AB2 + BC2 |By construction...(1)
But AC2 =AB2 + BC2 |Given ...(2)
 from (1) and (2),
PR2 =AC2
 PR = AC ...(3)
Now,
In ABC and PQR,
AB = PQ |By construction
BC = QR |By construction
AC = PR |By (3)
 ABC PQR |SSS congruence criterion
 B = Q |CPCT
But Q = 90° |By construction
 B = 90°
26. Construct an isosceles triangle whose base is 8 cm and altitude 4 cm and then another triangle
3
whose sides are times the corresponding sides of the isosceles triangle.
4
Sol. Required: To construct an isosceles triangle whose base is 8 cm and altitude 4 cm and then another
3
triangle whose sides are times the corresponding sides of the isosceles triangle.
4
Steps of construction:
1. Draw BC = 8 cm
2. Draw perpendicular bisector of BC. Let it meet BC at D.
3. Mark a point A on the perpendicular bisector such that AD = 4 cm.

Click Here to Buy Main Book Online


M-56 CBSE BOARD EXAMINATION PAPER–2019

A′
4 cm

B C′ C
8 cm D
B1

B2

B3
B4

4. Join AB and AC.


Then, ABC is the required isosceles triangle.
5. Draw any ray BX making an acute angle with BC.
6. Locate 4 points B1, B2, B3 and B4 on BX such that BB1 = B1B2 = B2B3 = B3B4.
7. Join B4C. Draw a line through B3 parallel to B4C intersecting BC at C.
8. Draw a line through C parallel to the line CA to intersect BA at A.
Then, ABC is the required triangle.
27. A boy standing on a horizontal plane finds a bird flying at a distance of 100 m from him at an
elevation of 30. A girl standing on the roof of a 20 m high building, find the elevation of the same
bird to be 45. The boy and the girl are on the opposite sides of the bird. Find the distance of the
bird from the girl. (Given 2 = 1.414)
OR
The angle of elevation of an aeroplane from a point A on the ground is 60. After a flight of
30 seconds, the angle of elevation changes to 30. If the plane is flying at a constant height of 3600
3 metres, find the speed of the aeroplane.
Sol.

P (Bird)

0m
10
D 45° B (Girl) roof

20 m Building
30°
A C
(Boy) E
Horizontal plane

Click Here to Buy Main Book Online


MATHEMATICS–X M-57
In right triangle PEA,
PE 1 PE
sin 30° =  =
AP 2 100
 PE = 50 m
 PD + DE = 50 m
 PD + BC = 50 m
 PD + 20 m = 50 m
 PD = 50 m – 20 m = 30 m
In right triangle PDE,
PD
sin 45° =
PB
1 30
 =
2 PB
 PB = 30 2 = 30 × 1.414 = 42.42 m
Hence, the distance of the bird from the girl is 42.42 m.
OR
In right triangle PMA,

PM 3600 3
tan 60° =  3 =
AM AM
 AM = 3600 m

In right triangle QNA,

QN 1 3600 3
tan 30° =  =
AN 3 AN
 AN = 10800 m
 PQ = MN = AN – AM
= 10800 m – 3600 m = 7200 m
Time of flight from P to Q = 30 seconds

Click Here to Buy Main Book Online


M-58 CBSE BOARD EXAMINATION PAPER–2019
 Speed of the aeroplane

7200
= m/s = 240 m/s
30

18
= 240 × km/hr
5
= 48 × 18 km/hr = 864 km/hr
28. Find the values of frequencies x and y in the following frequency distribution table, if N = 100 and
median is 32.
Marks: 0–10 10–20 20–30 30–40 40–50 50–60 Total
No. of Students: 10 x 25 30 y 10 100

OR
For the following frequency distribution, draw a cumulative frequency curve (ogive) of ‘more than
type’ and hence obtain the median value.
Class: 0–10 10–20 20–30 30–40 40–50 50–60 60–70
Frequency: 5 15 20 23 17 11 9

Sol.
Marks (class) No. of students (Frequency) Cumulative Frequency
0 – 10 10 10
10 – 20 x 10 + x
20 – 30 25 35 + x
30 – 40 30 65 + x
40 – 50 y 65 + x + y
50 – 60 10 75 + x + y
Total 100

N = 100
 75 + x + y = 100
 x + y = 25 ...(1)
The median is 32, which lies in the class 30 – 40. So, 30 – 40 is the median class.
So, l = 30, f = 30, cf = 35 + x, h = 10
N 
  cf 
 Median = l +  2  ×h
 f 

 100 
 2  (35  x) 
⇒ 32 = 30 +   × 10
 30 
 

50  35  x 15  x
 2=  2=
3 3
 6 = 15 – x  x=9 ...(2)
Putting x = 9 in (1), we get
9 + y = 25  y = 16 ...(3)

Click Here to Buy Main Book Online


MATHEMATICS–X M-59
OR
More Than Type Distribution
Item Cumulative Frequency
More than 0 100
More than 10 95
More than 20 80
More than 30 60
More than 40 37
More than 50 20
More than 60 9

We mark the lower limits of the class intervals on the horizontal axis (x-axis) and their corresponding
cumulative frequencies on the vertical axis (y-axis). Then, we plot the points (0, 100), (10, 95), (20, 80),
(30, 60), (40, 37), (50, 20) and (60, 9) on a graph paper and join them by a free hand smooth curve. The curve
we get is an ogive of more than type.
To obtain the median value

N 100
Locate = = 50 on the y-axis from this point, draw a line parallel to x-axis cutting the curve at a point.
2 2
From this point draw a perpendicular to the x-axis. The point of intersection of this perpendicular with the
x-axis determines 34.5, which is the median of the given data.
Thus, the median value is 34.5.

(0,100)
100
(10,95) More than type ogive
90

80 (20,80)

70
Cumulative frequency

60 (30,60)

50

40
(40,37)

30

20 (50,20)

10 (60,9)
Median (34.5)

X
0 10 20 30 40 50 60

Lower limits

Click Here to Buy Main Book Online


M-60 CBSE BOARD EXAMINATION PAPER–2019
29. Prove that:
(1 + cot  + tan  )(sin  – cos  )
= sin2  cos2 
(sec3  – cosec3  )

(1  cot   tan ) (sin   cos )


Sol. LHS =
sec3   cosec3 

 cos  sin  
1    (sin   cos )
=  sin  cos  
1 1

cos3  sin 3 

 sin  cos   cos2   sin 2  


  (sin   cos )
 sin  cos  
=
sin 3   cos3 
sin3  cos3 

(sin  cos   1) (sin   cos ) sin 3  cos3 


= 
sin  cos  sin 3   cos3 
| sin2 + cos2 = 1
(sin  cos   1) (sin   cos ) sin 2  cos2 
=
(sin   cos ) (sin 2   cos2   sin  cos )
(1  sin  cos ) sin 2  cos2 
= | sin2 + cos2 = 1
1  sin  cos 
= sin2 cos2= RHS
30. An open metallic bucket is in the shape of a frustum of a cone. If the diameters of the two circular
ends of the bucket are 45 cm and 25 cm and the vertical height of the bucket is 24 cm, find the area
of the metallic sheet used to make the bucket. Also find the volume of the water it can hold.
22
(Use  = )
7
Sol. Here, h = 24 cm
45
r1 = cm
2
25
r2 = cm
2
 Slant height (l) = h2  (r1  r2 )2
2
 45 25 
= (24)2    
 2 2 

= (24)2  (10)2 = 576  100


= 676 = 26 cm
Area of the metallic sheet used to make the bucket
= Total surface area of the bucket
= l (r1 + r2) + r22

Click Here to Buy Main Book Online


MATHEMATICS–X M-61
22  45 25  22 25 25
= × 26 ×    + × 
7  2 2  7 2 2
22 11  625
= × 26 × 35 +
7 14
6875
= 22 × 26 × 5 +
14
= 110 × 26 + 491.07
= 2860 + 491.07 = 3351.07 cm2
Also, volume of the water it can hold
1
=  h (r12  r22  r1 r2 )
3
1 22  45 2  25 2  45   25  
=   24        
3 7  2   2   2   2  
44
= {2025 + 625 + 1125}
7
44  3775 166100
= 
7 7
= 23728.57 cm 3

Click Here to Buy Main Book Online


MRP: 575.00

Click Here to Buy the Book Online

Committed to Educate the Nation NEW AGE INTERNATIONAL (P) LIMITED, PUBLISHERS
LONDON • NEW DELHI • NAIROBI

IN INDIA Visit us at www.newagegolden.com


M-62 CBSE BOARD EXAMINATION PAPER–2018

Series: TYM Set-2

Code No. 30/2


CBSE BOARD EXAMINATION PAPER-2018
Mathematics–X (Solved)

Time Allowed: 3 Hours Maximum Marks: 80


General Instructions:
(i) All questions are compulsory.
(ii) The question paper consists of 30 questions divided into four sections – A, B, C and D.
(iii) Section A contains 6 questions of 1 mark each, Section B contains 6 questions of 2 marks each, Section C contains 10 questions of 3
marks each, Section D contains 8 questions of 4 marks each.
(iv) There is no overall choice. However, an internal choice has been provided in four questions of 3 marks each and 3 questions of 4 marks
each. You have to attempt only one of the alternatives in all such questions.
(v) Use of calculators is not permitted.

SECTION–A
Question numbers 1 to 6 carry 1 mark each.
AB 1 ar Δ ABC
1. Given  ABC ~  PQR, if = , then find (1)
PQ 3 ar  PQR
Sol.  ABC ~  PQR (Given)
2
ar  ABC  AB 
 ar  PQR =  PQ 
 The ratio of the areas of two similar triangles
is equal to the square of the ratio of their
corresponding sides (½)
2 AB 1
1  =
=   PQ 3
3
1
= (½)
9
2. What is the value of (cos267° – sin223°)? (1)
Sol. (cos267° – sin223°)
= cos2 (90° – 23°) – sin2 23° (½)
= sin2 23° – sin2 23°  cos(90° – ) = sin 
=0 (½)
3. Find the distance of a point P(x, y) from the origin. (1)
Sol. Let O be the origin. Then, (½)
O  (0, 0)
P  (x, y)
Distance of point P from the origin

M-62

Click Here to Buy Main Book Online


MATHEMATICS–X M-63

= ( x  0) 2  ( y  0) 2  Using distance formula

= x2  y 2 (½)
4. If x = 3 is one root of the quadratic equation x2 – 2kx – 6 = 0, then find the value of k. (1)
Sol. If x = 3 is one root of the quadratic equation x2 – 2kx – 6 = 0, then
(3)2 – 2k(3) – 6 = 0 (½)
 9 – 6k – 6 = 0
 – 6k + 3 = 0
 6k = 3
3
 k=
6
1
 k= (½)
2
1
Hence, the required value of k is .
2
5. What is the HCF of smallest prime number and the smallest composite number? (1)
Sol. The smallest prime number = 2
The smallest composite number = 4 = 2 × 2 = 22 (½)
 Required HCF = 2 (½)
6. In an AP, if the common difference (d) = – 4 and the seventh term (a7) is 4, then find the first term. (1)
Sol. d=–4
a7 = 4
Let the first term be a.
We know that
an = a + (n – 1)d (½)
Put n = 7
a7 = a + (7 – 1)d
 a7 = a + 6d
 4 = a + 6(– 4)
 4 = a – 24
 a = 4 + 24 = 28 (½)
Hence the first term is 28.

SECTION–B
Question numbers 7 to 12 carry 2 marks each.
7. An integer is chosen random between 1 and 100. Find the probability that it is:
(i) divisible by 8. (ii) not divisible by 8. (2)
Sol. Integers between 1 and 100 are
2, 3, 4, …, 99
Their numbers = 98
 Number of all possible outcomes = 98
(i) Let E be the event that the integer is divisible by 8.
Then, the outcomes favourable to E are 8, 16, 24, 32, 40, 48, 56, 64, 72, 80, 88 and 96.
Therefore, the number of outcomes favourable to E = 12
 P(E) = P(divisible by 8)

Click Here to Buy Main Book Online


M-64 CBSE BOARD EXAMINATION PAPER–2018

Number of outcomes favourable to E


= Number of all possible outcomes
12 6
=  (1)
98 49
(ii) Let F be the event that the integer is not divisible by 8. Then,
P(F) = 1 – P(E)
6
=1–
49
43
= (1)
49
8. Two different dice are tossed together. Find the probability
(i) of getting a doublet
(ii) of getting a sum 10, of the numbers on the two dice. (2)
Sol. Number of all possible outcomes when two different dice are tossed together = 6 × 6 = 36.
(i) Let E be the event of getting a doublet. Then the number of outcomes favourable to E {(1, 1), (2, 2), (3, 3), (4, 4), (5, 5) and
(6, 6)} = 6
Number of outcomes favourable to E
 P(E) = Number of all possible outcomes
6 1
=  (1)
36 6
(ii) Let F be the event of getting a sum 10, of the numbers on the two dice. Then, the number of outcomes favourable to
F{(4, 6), (6, 4), and (5, 5)} = 3
Number of outcomes favourable to F
 P(F) = Number of all possible outcomes

3 1
=  (1)
36 12
9. Find the ratio in which P(4, m) divides the line segment joining the points A(2, 3) and B(6, – 3). Hence find m. (2)
Sol. We have,
A  (2, 3)
B  (6, – 3)
P  (4, m)
Let P divide the line segment AB in the ratio k : 1. Then by
k 1
A B
(2, 3) P (6, –3)
(4, m)

section formula,
 ( k )(6)  (1)(2) ( k )(  3)  (1)(3) 
P  , 
 k 1 k 1 

 6k  2  3k  3 
 P  ,
k  1 
(½)
 k 1
But P  (4, m)
6k  2
 =4 …(1)
k 1

Click Here to Buy Main Book Online


MATHEMATICS–X M-65

 3k  3
and =m …(2)
k 1
From (1) 6k + 2 = 4(k + 1)
 6k + 2 = 4k + 4
 6k – 4k = 4 – 2
 2k = 2
2
 k= =1 …(3) (1)
2
Hence, the required ratio is 1 : 1.
3 (1)  3
Again, from (2), m= |Using (3)
11
 m= 0
Hence, the required value of m is 0. (½)

10. Given that 2 is irrational, prove that  5 + 3 2  is an irrational number. (2)

Sol.  3 is a rational number and 2 is an irrational number (given)

 3 2 is an irrational number
 The product of a non-zero rational
number and an irrational number is an
irrational number. …(1) (1)
Now,  5 is a rational number and 3 2 is an irrational number [by (1)]
  5  3 2  is an irrational number.
 The sum of a rational number and an
irrational number is an irrational number.
(1)
11. In Fig., ABCD is a rectangle. Find the values of x and y. (2)
x+y
S C

14 cm x–y

A B
30 cm
Sol.  ABCD is a rectangle
 AB = DC  opposite sides of a rectangle are equal
and AD = BC (½)
 30 = x + y
14 = x – y
 x + y = 30 …(1)
x – y = 14 …(2) (½)
Adding equation (1) and equation (2), we get
2x = 44
44
 x= = 22 (½)
2
Subtracting equation (2) from equation (1), we get
2y = 16
16
 y= =8 (½)
2
Hence, the required values of x and y are 22 and 8 respectively.

Click Here to Buy Main Book Online


M-66 CBSE BOARD EXAMINATION PAPER–2018

12. Find the sum of first 8 multiples of 3.


Sol. First 8 multiples of 3 are
3, 6, 9, 12, 15, 18, 21, and 24
Here a2 – a1 = 6 – 3 = 3
a3 – a2 = 9 – 6 = 3
a4 – a3 = 12 – 9 = 3
i.e., ak+1 – ak is the same every time.
So, the above list of number forms an AP. (½)
Here, a= 3
d= 3
n= 8 (½)
 Sum of first 8 multiples of 3 = S8
8
= [2a + (8 – 1)d]
2
n
 Sn = [2a + (n – 1)d]
2
= 4[2 × 3 + 7 × 3] = 4[6 + 21]
= 4 × 27 = 108 (1)

SECTION–C
Question numbers 13 to 22 carry 3 marks each.
13. A plane left 30 minutes late than its scheduled time and in order to reach the destination 1500 km away in time, it had to
increase its speed by 100 km/h from the usual speed. Find its usual speed. (3)
Sol. Let the usual speed of the plane be x km/h. Then,

1500 Distance
the time taken by the plane to reach the destination 1500 km away at the usual speed = h  Time = (½)
x Speed
Increased speed of the plane = (x + 100) km/h
1500 Distance
Then, the time taken by the plane to reach the destination at the increased speed = h  Time = (½)
x  100 Speed

According to the question,


500 1500 1 30 1
 = 30 minutes = h  h (½)
x x  100 2 60 2
1 1  1
 1500   =
 x x  100  2
1 1 1
  =
x x  100 2  1500
( x  100)  x 1
 =
x( x  100) 3000
100 1
 2 =
x  100 x 3000
 x2 + 100x = 300000  Cross-multiplying

Click Here to Buy Main Book Online


MATHEMATICS–X M-67

 x2 + 100x – 300000 = 0
 x2 + 600x – 500x – 300000 = 0
 x(x + 600) – 500(x + 600) = 0
 (x + 600) (x – 500) = 0
 x + 600 = 0 or x – 500 = 0
 x = – 600 or x = 500
 x = – 600, 500 (1)
x = – 600 is inadmissible as x is the speed of the plane and speed cannot be negative.
 x = 500 (½)
Hence the usual speed of the plane is 500 km/h.
14. Prove that the area of an equilateral triangle described on one side of the square is equal to half the area of the equilateral
triangle described on one of its diagonal. (3)
OR
If the area of two similar triangles are equal, prove that they are congruent.
Sol. Given: ABCD is a square whose one diagonal is AC.  APC and  BQC are two equilateral triangles described on the diagonal
AC and side BC of the square ABCD respectively.
1
To Prove: ar ( BQC) = ar ( APC)
2
P
B
A

D C (1)
Proof:   APC and  BQC are both equilateral triangles
  APC ~  BQC
 By AAA similarity criterion
ar ( APC)  AC 
2
 ar ( BQC) =   (1)
 BC 
 The ratio of the areas of two similar
triangles is equal to the square of the ratio
of their corresponding sides
2
 
=  2BC   Diagonal = 2 side
 BC 
=2
1
 ar ( BQC) = ar ( APC) (1)
2
OR
Given: ABC and DEF are two similar triangles such that
ar ( ABC) = ar ( DEF)
To Prove:  ABC   DEF A D

B C E F

Click Here to Buy Main Book Online


M-68 CBSE BOARD EXAMINATION PAPER–2018

Proof:  ABC ~  DEF  Given


ar ( ABC)  BC 
2
 ar ( DEF) =  
 EF 
 The ratio of the areas of two similar triangles
is equal to the square of the ratio of their
corresponding sides.
2
 BC 
 1=  
 EF 
  ar ( ABC) = ar ( DEF) (Given)
BC 2
 1=
EF2
 BC = EF2
2

 BC = EF …(1) (2)
Also,  ABC =  DEF ...(2)
and  ACB =  DFE ...(3)
  ABC ~  DEF [Corresponding angles
of two similar triangles are equal]
From (1), (2) and (3)
 ABC   DEF  By ASA congruence criterion (1)
15. Prove that the lengths of tangents drawn from an external point to a circle are equal. (3)
Sol. Given: A circle with centre O, a point P lying outside the circle and two tangents PQ, PR on the circle from P.
To Prove: PQ = PR
Construction: Join OP, OQ and OR.
Proof: OQP = 90°
 The tangent at any point of a circle is
perpendicular to the radius through the
point of centre. (½)
Q

P O

(1)
R

 ORP = 90°
 The tangent at any point of a circle is
perpendicular to the radius through the point
of contact (½)
Now, in right triangles OQP and ORP,
OQ = OR | Radii of the same circle.
OP = OP | Common
  OQP   ORP | By RHS congruence criterion
 PQ = PR | CPCT (1)

Click Here to Buy Main Book Online


MATHEMATICS–X M-69

16. A wooden article was made by scooping out a hemisphere from each end of a solid cylinder, as shown in Fig. If the height of
the cylinder is 10 cm and its base is of radius 3.5 cm. Find the total surface area of the article.

OR
A heap of rice is in the form of a cone of base diameter 24 m and height 2.5 m. Find the volume of the rice. How much canvas
cloth is required to just cover the heap? (3)
Sol. 1. For hemispherical end (1)
1
35 7
Radius (r) = 3.5 cm =  cm
10 2
 Surface area 3 10 cm
= 2r2
2
22  7  2
= 2.  
7 2
= 77 cm2 3.5 cm…(1) (1)
2. For hemispherical end (2)
7
Radius (r) = cm
2
 Surface area = 77 cm2 …(2) (½)
3. For cylindrical portion (3)
7
Radius (r) =
2
Height (h) = 10 cm
 Surface area = 2rh
22 7
= 2. . .10 = 220 cm2 …(3) (1)
7 2
 Total surface area of the article
= Surface area of the hemispherical end (1) + Surface area of the hemispherical end (2) + Surface area
of the cylindrical portion (3)
= (1) + (2) + (3)
= 77 cm2 + 77 cm2 + 220 cm2
= 374 cm2 (½)
OR
For cone
Base diameter = 24 m
24
 Base radius (r) = m = 12 m
2

3.5 m

24 m

Click Here to Buy Main Book Online


M-70 CBSE BOARD EXAMINATION PAPER–2018

35 7
Height (h) = 3.5 m = m m
10 2
1 2
 Volume = r h
3
2
1 22  7 
= . .  .(12)
3 7 2
= 154 m3
Hence, the volume of the rice is 154 m3. (1)

Again,Slant height (l) = r 2  h2

2
7 49
= (12) 2    = 144 
2 4

576  49 625 25
= =  m (1)
4 4 2
 Curved surface area = rl
22 7 25 275
= . . 
7 2 2 2
= 137.5 m2 (1)
Hence, 137.5 m2 of canvas cloth is required to just cover the heap.
17. The table below shows the salaries of 280 persons:
Salary (In thousand `) (No. of persons)
5–10 49
10–15 133
15–20 63
20–25 15
25–30 6
30–35 7
35–40 4
40–45 2
45–50 1

Calculate the median salary of the data. (3)

Sol. Salary (In thousand `) (No. of persons) Cumulative frequency


5–10 49 49
10–15 133 182
15–20 63 245
20–25 15 260
25–30 6 266
30–35 7 273
35–40 4 277
40–45 2 279
45–50 1 280
(1)

Click Here to Buy Main Book Online


MATHEMATICS–X M-71

Now, n = 280
n 280
So, = = 140
2 2
This observation lies on the class 10-15. Therefore, 10-15 is the median class.
So, l = 10
cf = 49
f = 133
h= 5 (1)

n 
  cf 
 Median = l +  2  ×h
 f 

 140  49 
= 10 +   ×5
 133 

91  5
= 10 +
133

455
= 10 +
133
= 10 + 3.42 = ` 13.42 (1)
Hence, the median salary of the data is ` 13.42.
4 sin θ  cos θ + 1 
18. If 4 tan  = 3, evaluate  .
 4 sin θ + cos θ  1 
OR
If tan 2A = cot (A – 18°), where 2A is an acute angle, find the value of A.
Sol. Let us draw a right triangle ABC in which  BAC = 
4 tan  = 3
3 C
 tan  =
4

BC 3
 = 5k 3k
AB 4

BC AB
 = = k (say) A
q
B
3 4 4k
where k is a positive number
 BC = 3k
AB = 4k (½)
By using the Pythagoras theorem, we have
AC2 = AB2 + BC2
= (4k)2 + (3k)2
= 16k2 + 9k2
= 25k2
 AC = 25k 2
 AC = 5k (½)

Click Here to Buy Main Book Online


M-72 CBSE BOARD EXAMINATION PAPER–2018

Therefore,
BC 3k 3
sin  = = = (½)
AC 5k 5
AB 4k 4
and cos  = = = (½)
AC 5k 5
 4 sin   cos   1 
  4 sin   cos   1 
 
3 4
4
 1
= 5 5
3 4
4  1
5 5
12 4 8
 1 1
5
= 5 5 =
12 4 16
 1 1
5 5 5
13
13
= 5  (1)
11 11
5
OR
tan 2 A = cot (A – 18°)
 tan 2 A = tan {90°– (A – 18°)}
| tan (90° – ) = cot 
 tan 2A = tan (108°– A) (1½)
 2A = 108°– A
 2A + A = 108°
 3A = 108°
108
 A= = 36° (1½)
3
19. Find the area of the shaded region in Fig., where arcs drawn with centers A, B, C and D intersect in pairs at mid-points P,
Q, R and S of the sides AB, BC, CD and DA respectively of a square ABCD of side 12 cm. [Use  = 3.14] (3)
P
A B

S Q

D C
R
P
Sol. AB = BC = CD = DA = 12 cm A B
 Area of square ABCD I II
= (12)2 = 144 cm2 (1) Q
S
For sector I, i.e., quadrant APSA
IV III
1
r = AP = AB D C
2 R

 P is the mid point of AB

Click Here to Buy Main Book Online


MATHEMATICS–X M-73

1
= × 12 = 6 cm
2
 = 90°

 Area = r2
360
90
= (6)2 = 9 cm2
360
Similarly,
Area of sector II (i.e., quadrant BPQB) = 9 cm2
Area of sector III (i.e., quadrant CQRC) = 9 cm2
Area of sector IV (i.e., quadrant DRSD) = 9 cm2 (1)
 Area of the shaded region
= Area of square ABCD – [Area of sector I
+ Area of sector II
+ Area of sector III
+ Area of sector IV]
= 144 – (9 + 9 + 9 + 9)
= 144 – 36
= 144 – 36 × 3.14
= 144 – 113.04
= 30.96 cm2 (1)
20. If A(– 2, 1), B(a, 0), C(4, b) and D(1, 2) are the vertices of a parallelogram ABCD, find the values of a and b. Hence find the
lengths of its sides.
OR
If A(– 5, 7), B(– 4, – 5), C(– 1, – 6) and D(4, 5) are the vertices of a quadrilateral ABCD, find the area of the quadrilateral
ABCD.
(3)
Sol. Let the diagonals AC and BD of the parallelogram ABCD intersect at E. D (1, 2) C (4, b)
Then, E is the mid-point of AC as well as BD.

Diagonals of a parallelogram
bisect each other E

 Mid-point of AC = Mid-point of BD

  2  4 1 b   a 1 0  2
  , =  ,  | By mid-point formula A (–2,1) B (a, 0)
 2 2   2 2 
(1)

 1 b   a 1 
  1,  =  , 1
 2   2 

a 1
 1= ,
2

1 b
= 1
2
 a + 1 = 2, b + 1 = 2
 a = 1, b = 1

Click Here to Buy Main Book Online


M-74 CBSE BOARD EXAMINATION PAPER–2018

Hence, the values of a and b are 1 and 1 respectively.


Now,
A  (– 2, 1)
B  (1, 0)
C  (4, 1)
D  (1, 2)

 AB = {1  (  2)}2  (0  1) 2  By distance formula

= 9  1  10 units

 BC = (4  1) 2  (1  0) 2  By distance formula

= 9  1  10 units

 CD = (1  4) 2  (2  1) 2  By distance formula

= 9  1  10 units

 DA = ( 2  1) 2  (1  2) 2  By distance formula

= 9  1  10 units

Hence, the lengths of its sides are 10 units each. (1)


OR
Join BD.
Area of triangle ABD
1
= [– 5(– 5 – 5) – 4(5 – 7) + 4{7 – (– 5)}]
2
1
= [50 + 8 + 48]
2
1
= (106) = 53 sq units …(1) (1)
2
C (–1, –6)

D (4, 5)

A (–5, 7) B (–4, –5)

Area of triangle BCD


1
= [(– 4)(– 6 – 5) + (– 1) {5 – (– 5)} + 4{– 5 – (– 6)}]
2
1
= [44 – 10 + 4]
2
1
= (38) = 19 square units …(2) (1)
2

Click Here to Buy Main Book Online


MATHEMATICS–X M-75

Now, Area of the quadrilateral ABCD


= Area of triangle ABD + Area of triangle BCD
= 53 sq units + 19 sq units  from (1) and (2)
= 72 sq units (1)
21. Find HCF and LCM of 404 and 96 and verify that HCF × LCM = Product of the two given numbers. (3)
Sol. 404

2 202

2 101

 404 = 2 × 2 × 101 = 22 × 101

96

2 48

2 24

2 12

2 6

2 3

 96 = 2 × 2 × 2 × 2 × 2 × 3 = 25 × 3
Therefore,
HCF (404, 96) = 22 = 4 (1)
LCM (404, 96) = 25 × 3 × 101
= 32 × 3 × 101
= 96 × 101 = 9696 (1)
Verification
HCF × LCM = 4 × 9696 = 38784 …(1)
Product of two given numbers
= 404 × 96
= 38784 …(2)
From (1) and (2), we get
HCF × LCM = Product of the two given numbers (1)

22. Find all zeroes of the polynomial (2x4 – 9x3 + 5x2 + 3x – 1) if two of its zeroes are  2 + 3  and  2  3  . (3)
Sol. Let p(x) = 2x4 – 9x3 + 5x2 + 3x – 1

Since two zeroes of p(x) are  2  3  and  2  3  , therefore

{x –  2  3  } {x –  2  3  }

Click Here to Buy Main Book Online


M-76 CBSE BOARD EXAMINATION PAPER–2018

i.e.,{(x – 2) – 3 } {(x – 2) + 3}

i.e., (x – 2)2 –  3 2
i.e., x2 – 4x + 4 – 3
i.e., x2 – 4x + 1
is a factor of p(x). (1)
Now we apply the division algorithm to p(x) and x2 – 4x + 1
2
2x – x – 1
4 3 2
x2 – 4 x + 1 2x – 9x + 5x + 3x – 1
4 3 2
2x – 8x + 2x
– + –
3 2
– x + 3x + 3x – 1
3 2
– x + 4x – x
+ – +
2
– x + 4x – 1
2
– x + 4x – 1
+ – +
0

So, p(x) = (x2 – 4x + 1) (2x2 – x – 1) (1)


 Other two zeroes are given by
2x2 – x – 1 = 0
 2x2 – 2x + x – 1 = 0
 2x(x – 1) + 1(x – 1) = 0
 (x – 1) + (2x + 1) = 0
 x – 1 = 0 or 2x + 1 = 0
1
 x = 1 or x = 
2
1
 x= 1,  (1)
2
1
 Other two zeroes are 1 and  .
2
1
Hence, all the zeroes of the given polynomial are 2  3 , 2  3 , 1 and  . (1)
2

SECTION–D
Question numbers 23 to 30 carry 4 marks each.
3
23. Draw a triangle ABC wih BC = 6 cm, AB = 5 cm and  ABC = 60°. Then construct a triangle whose sides are of the
4
corresponding sides of the  ABC. (4)
Sol. Required: To draw a triangle ABC with sides BC = 6 cm, AB = 5 cm and  ABC = 60° and then to construct a triangle whose
3
sides are of the corresponding sides of the  ABC.
4

Click Here to Buy Main Book Online


MATHEMATICS–X M-77

Steps of Construction: A
(1)
1. Draw a triangle with side BC = 6 cm, AB = 5 cm and  ABC = 60°. A¢
2. Draw any ray BX making an acute angle with BC on the side opposite to the vertex A.
5 cm
3. Locate 4 points B1, B2, B3 and B4 on BX such that BB1 = B1B2 = B2B3 = B3B4. (1)
4. Join B4 C and draw a line through B3 parallel to B4 C intersecting BC at C. 60°
B C
5. Draw a line through C parallel to the line CA to intersect BA at A. Then  ABC is the 6 cm C¢
required triangle. (3)
B1
24. The sum of four consecutive numbers in an AP is 32 and the ratio of the product of the
B2
first and the last term to the product of two middle terms is 7 : 15. Find the numbers.
(4) B3
Sol. Let the four consecutive terms in an AP be a – 3d, a – d, a + d, and a + 3d.
B4
Then, according to the question
X
(a – 3d) + (a – d) + (a + d) + (a + 3d) = 32
 4a = 32
 a= 8 …(1) (1)
(a  3d ) (a  3d ) 7
and, =
( a  d ) (a  d ) 15

a 2  9d 2 7
 2 2 = (1)
a d 15
 15(a2 – 9d2) = 7(a2 – d2) | Cross-multiplying
 15a2 – 135d2 = 7a2 – 7d2
 15a2 – 7a2 = 135d2 – 7d2
 8a2 = 128d2
 a2 = 16d2  dividing throughout by 8
 (8)2 = 16d2 | From (1)
 64 = 16d2
 16d2 = 64
64
 d2 = =4
16
 d= 4 =±2 (1)
Case I when d = 2, then the four numbers are
8 – 3(2), 8 – 2, 8 + 2, 8 + 3(2)
or, 2, 6, 10, 14 (½)
Case II when d = – 2, then the four numbers are
8 – 3(– 2), 8 – (– 2), 8 + (– 2), 8 + 3(– 2)
or, 14, 10, 6, 2 (½)
Hence the required numbers are
2, 6, 10 and 14 or 14, 10, 6 and 2.
1
25. In an equilateral  ABC, D is a point on side BC such that BD = BC . Prove that 9(AD)2 = 7(AB)2.
3
OR
Prove that, in a right triangle, the square on the hypotenuse is equal to the sum of the squares on the other two sides. (4)
1
Sol. Given: In an equilateral triangle ABC, D is a point on the side BC such that BD = BC.
3
To Prove: 9(AD)2 = 7(AB)2

Click Here to Buy Main Book Online


M-78 CBSE BOARD EXAMINATION PAPER–2018

B C
D E

Construction: Draw AE  BC (1)


Proof: In right triangles AEB and AEC,
Hyp. AB = Hyp. AC
  ABC is an equilateral triangle  AB = BC = CA
AE = AE  Common side
  AEB   AEC  By RHS congruence criterion
 BE = CE  CPCT
1
 BE = CE = BC …(1) (1)
2
In right triangle AED,
AD2 = AE2 + DE2 | By Pythagoras Theorem
= AE2 + (BE – BD)2
2
1 1 
= AE2 +  BC  BC 
2 3 
2
1 
= AE2 +  BC 
6 
1
= AE2 + BC2 …(2) (1)
36
In right triangle AEB,
AB2 = AE2 + BE2
2
1 
 AB2 = AE2 +  BC 
2 
1
 BC2 = AE2 + BC2   AB = BC = CA
4
3
 AE2 = BC2 …(3) (½)
4
From (2) and (3)
3 1
AD2 = BC2 + BC2
4 36
28 7
= BC2  BC2
36 9
7
= AB2   AB = BC = CA
9
 9AD2 = 7AB2 (½)

Click Here to Buy Main Book Online


MATHEMATICS–X M-79

OR
Given: A right triangle ABC right angled at B.
To Prove: AC2 = AB2 + BC2
Construction: Draw BD  AC
B

A C
D (1)
Proof: In  ADB and  ABC
 ADB =  ABC (Each = 90°)
 BAD =  CAB | Common angle
  ADB ~  ABC | By AA criterion of similarity

AD AB
 =
AB AC
 Corresponding sides of two similar
triangles are proportional
 AD.AC = AB2 …(1) (1)
Again, in  BDC and  ABC,
 BDC =  ABC (Each 90°)
 BCD =  ACB | Common angle
  BCD ~  ABC  By AA criterion of similarity

CD BC
 =
BC AC
 Corresponding sides of two similar
triangles are proportional
 CD.AC = BC2 …(2) (1)
Adding (1) and (2)
AD.AC + CD.AC = AB2 + BC2
 AC(AD + CD) = AB2 + BC2
 AC.AC = AB2 + BC2
 AC2 = AB2 + BC2 (1)
26. A motor boat whose speed is 18 km/hr in still water takes 1 hr more to go 24 km upstream than to return downstream to the
same spot. Find the speed of the stream.
OR
A train travels at a certain average speed for a distance of 63 km and then travels at a distance of 72 km at an average speed
of 6 km/hr more than its original speed. If it takes 3 hours to complete total journey, what is the original average speed?
(4)
Sol. Let the speed of the stream be x km/hr.
Speed of the motor boat in still water = 18 km/hr
Speed of the motor boat in downstream = (18 + x) km/hr (½)
Speed of the motor boat in upstream = (18 – x) km/hr (½)

Click Here to Buy Main Book Online


M-80 CBSE BOARD EXAMINATION PAPER–2018

24
Time taken by the motor boat to go 24 km downstream = hr
18  x
Distance
 Time = (½)
Speed
24
Time taken by the motor boat to go 24 km upstream = hr
18  x
Distance
 Time =
Speed (½)
According to the question,
24 24
 =1 (½)
18  x 18  x
 1 1 
 24   =1
 18  x 18  x 
1 1 1
  =
18  x 18  x 24
(18  x)  (18  x) 1
 =
(18  x) (18  x) 24
2x 1
 2 =
324  x 24
 324 – x2 = 48x | Cross multiplying
 x2 + 48x – 324 = 0
 x2 + 54x – 6x – 324 = 0
 x(x + 54) – 6(x + 54) = 0
 (x + 54) (x – 6) = 0
 x + 54 = 0 or x – 6 = 0
 x = – 54 or x=6
 x = – 54, 6 (1)
x = – 54 is inadmissible as x is the speed and the speed cannot be negative.
 x= 6 (½)
Hence, the speed of the stream is 6 km/hr.
OR
Let the original average speed of the train be x km/hr.
Then, the average speed of 6 km/hr more than its original speed = (x + 6) km/hr (½)

63 Distance
Time taken by the train to travel a distance of 63 km at the original average speed = hr  Time =
x Speed (½)
Time taken by the train to travel a distance of 72 km at an average speed of 6 km/hr more than its original speed
72 Distance
= x  6 hr  Time =
Speed (½)
According to the question,
63 72
 =3 (½)
x x6

7 8 
 9  =3
 x x  6 

Click Here to Buy Main Book Online


MATHEMATICS–X M-81

7 8 
 3   =1
 x x 6
7 8 1
  =
x x6 3
7( x  6)  8 x 1
 =
x( x  6) 3
15 x  42 1
 2
=
x  6x 3
 x 2 + 6x = 3(15x + 42) | Cross multiplying
 x 2 + 6x = 45x + 126
 x 2 – 39x – 126 = 0
 x 2 – 42x + 3x – 126 = 0
 x(x – 42) + 3(x – 42) = 0
 (x – 42) (x + 3) = 0
 x – 42 = 0 or x + 3 = 0
 x = 42 or x=–3
 x = 42, – 3
x = – 3 is inadmissible as x is the speed and the speed cannot be negative.
 x = 42 (2)
Hence, the original average speed of the train is 42 km/hr.
27. As observed from the top of a 100 m high light house from the sea-level, the angles of depression of two ships are 30° and
45°. If one ship is exactly behind the other on the same side of the light house, find the distance between two ships.
[Use 3 = 1.732] (4)
Sol. In right triangle, ABQ (1)

AB
tan 45° =
BQ

100
 1=
BQ

 BQ = 100 m …(1) (1)

A X
30° 45°
Light house
100 m

45° 30°
B P
Q (Ship)
(Ship)

In right triangle ABP,

AB
tan 30° =
BP

1 100
 =
3 BP

Click Here to Buy Main Book Online


M-82 CBSE BOARD EXAMINATION PAPER–2018

 BP = 100 3
= 100 × 1.732 = 173.2 m …(2) (1)
Therefore distance between the two ships
= PQ
= BP – BQ
= 173.2 – 100 = 73.2 m (1)
Hence, the distance between the two ships is 73.2 m.
28. The diameters of the lower and upper ends of a bucket in the form of a frustum of a cone are 10 cm and 30 cm respectively.
If its height is 24 cm, find:
(i) The area of the metal sheet used to make the bucket.
(ii) Why we should avoid the bucket made by ordinary plastic?
[Use  = 3.14] (4)
Sol. (i) Here, (½)
30
r1 = cm = 15 cm
2
10
r2 = cm = 5 cm
2
h = 24 cm (½)
30 cm

24 cm

10 cm

 Slant height (l) = h 2  (r1  r2 )2

= (24) 2  (15  5) 2

= 576  100

= 676 = 26 cm (1)
 Total surface area of the bucket (excluding the upper end)
= l(r1 + r2) + r22
= (26) (15 + 5) + (5)2
= 520 + 25 = 545
= 545 × 3.14 = 1711.3 cm2 (1)
Hence, the area of the metal sheet used to make the bucket is 1711.3 cm2.
(ii) We should avoid the bucket made by ordinary plastic because it is not as strong as is required for practical purposes. By
filling an ordinary bucket by water or some other liquid, it cannot bear the weight of that water or liquid and therefore the
chances are even that the bucket may be broken. Thus, for safety and strength purpose, we should avoid the bucket
made by ordinary plastic. (1)

Click Here to Buy Main Book Online


MATHEMATICS–X M-83

29. The mean of the following distribution is 18. Find the frequency f of the class 19–21. (4)
Class 11-13 13-15 15-17 17-19 19-21 21-23 23-25
Frequency 3 6 9 13 f 5 4

OR
The following distribution gives the daily income of 50 workers of a factory:
Daily Income (in `) 100-120 120-140 140-160 160-180 180-200
Number of workers 12 14 8 6 10
Convert the distribution above to a less than type cumulative frequency distribution and draw its ogive.

Sol. Class Class Mark(xi) Frequency(fi) fixi

11–13 12 3 36
13–15 14 6 84
15–17 16 9 144
17–19 18 13 234
19–21 20 f 20f
21–23 22 5 110
23–25 24 4 93
Total fi = f + 40 fixi = 20f + 704

Using the direct method, (2)


fi xi
x = (1)
fi
20 f  704
 18 =
f  40
 20f + 704 = 18(f + 40)
 20f + 704 = 18f + 720
 20f – 18f = 720 – 704
 2f = 16
 f= 8 (1)
Hence, the frequency f of the class 19–21 is 8.
OR
Less than type cumulative frequency distribution

Daily income (in `) Cumulative frequency


Less than 120 12
Less than 140 12 + 14 = 26
Less than 160 26 + 8 = 34
Less than 180 34 + 6 = 40
Less than 200 40 + 10 = 50 (2)

Click Here to Buy Main Book Online


M-84 CBSE BOARD EXAMINATION PAPER–2018

50

‘Less than’
ogive
40
Cumulative Frequency

30

20

10

X
120 140 160 180 200
Upper Limits

(2)

sin A  2 sin 3 A
30. Prove that: = tan A
2 cos 3 A  cos A

sin A  2 sin 3 A
Sol. LHS =
2 cos3 A  cos A
sin A(1  2 sin 2 A)
= (1)
cos A(2 cos 2 A  1)
sin A(sin 2 A  cos 2 A  2 sin 2 A)
=
cos A(2 cos 2 A  sin 2 A  cos 2 A)
  sin2 A + cos2 A = 1 (2)
sin A(cos 2 A  sin 2 A)
=
cos A(cos 2 A  sin 2 A)
sin A
= = tan A
cos A
= RHS (1)

Click Here to Buy Main Book Online


Click Here to Buy Main Book Online
Strictly
Based on
Latest CLASS
NCERT X
Textbooks
REFERENCE BOOKS

MRP: 550.00 MRP: 355.00 MRP: 335.00


Click Here to Buy the Book Online Click Here to Buy the Book Online Click Here to Buy the Book Online

MRP: 255.00 MRP: 495.00 MRP: 598.00


Click Here to Buy the Book Online Click Here to Buy the Book Online Click Here to Buy the Book Online

MRP: 299.00 MRP: 240.00 MRP: 290.00


Click Here to Buy the Book Online Click Here to Buy the Book Online Click Here to Buy the Book Online

Committed to Educate the Nation NEW AGE INTERNATIONAL (P) LIMITED, PUBLISHERS
LONDON • NEW DELHI • NAIROBI

IN INDIA Visit us at www.newagegolden.com


MRP: 575.00

Click Here to Buy the Book Online

Committed to Educate the Nation NEW AGE INTERNATIONAL (P) LIMITED, PUBLISHERS
LONDON • NEW DELHI • NAIROBI

IN INDIA Visit us at www.newagegolden.com

You might also like